Sie sind auf Seite 1von 181

FOR THE LOVE OF THE CLINICAL ART

A Primer of Clinical Diagnosis & A Tribute to Emeritus Professor TJ Danaraj

Dr Wong Yin Onn,


MBBS (Mal), MRCP (UK), AM (Msia), FRCP (Glas)

Associate Professor, Monash University


Professor TJ Danaraj was the Foundation Dean of the Faculty of Medicine, University of Malaya. Those who have had the privilege of being taught by this giant in Malaysian medicine would have no doubt this was an extraordinary individual, an exemplary and dedicated teacher and phenomenal clinician. He grew up in colonial times to be a doctor, fighting discrimination and difficult times, he transformed a piece of empty land on a hill at the University of Malaya into a Temple of Medical Knowledge and service, and where he was the High Priest of Medical Education till he was 80 years old. Only failing health stopped him from teaching his beloved students. His life mission was a single mindedness quest for excellence in Medical Education. TJ Danaraj dedicated his whole lifetime towards medical education in Singapore and in Malaysia. After World War II he joined the teaching staff of the King Edward College of Medicine, which in 1949 became the Faculty of Medicine in the University of Malaya in Singapore. He was elected Dean of the Faculty in 1960. As Dean he introduced postgraduate medical education, made changes in the curriculum and increased the intake of students to produce more doctors sorely needed in the country. In 1963 newly independent Malaysia decided to set up its own medical school as the country was facing an acute shortage of doctors. TJ Danaraj was the obvious choice. He accepted the appointment as Foundation Dean and under his leadership, not only was the structure of the University of Malaya Medical Centre completed in record time but the first batch of doctors graduated in 1969, a mere six years after the decision was taken to build the medical school. His also helped established a second medical school in Saudi Arabia. TJ Danaraj fulfilled his life long passion of excellence in Medical Education to produce doctors of high caliber. Stories told by his students testify to the legacy he left behind and the extraordinary way he was able to mold and inspire them. Invariably the initial fear on first encounter with Prof Danaraj turned into respect, affection and friendship.

In 1941, T.J. Danaraj was a young doctor at the Butterworth Hospital. Positions in the colonial medical service were reserved for British doctors and "natives", no matter how qualified, could only serve as assistants. This became unbearable when local doctors found themselves more competent and hardworking than their British superiors. One day, during a Japanese air strike, he received a call from C.V. Norris, head of medical and health services in Province Wellesley, alerting him to send his wife to an evacuation point for women at the Penang wharf. But when his wife crossed the Penang channel and got there, she found that only British women and children were allowed in. Fortunately, a ferry was still operating and she managed to return to Butterworth. Like many of his contemporaries, events like this motivated him to subsequently devote his life to social service.

His MD thesis

This original work was part of Prof TJ Danaraj's MD thesis. The original slides of his work are now in the Singapore General Hospital Museum.

Autopsies and Clinical Skills It was the dogged pursuit of the truth that made the Clinicians of the old days attend diligently to the autopsies of their patients that the Art could not help; from the cold steel table, the final diagnosis could be known and clinical signs correlated. From such was the Clinical Art perfected. Today we have 'digital autopsies' and the patients are still alive; I beg my students to correlate the ECHO or the CT or the MRI to their clinical findings to improve their art of clinical diagnosis. Osler did his autopsies with his bare hands and had cutaneous TB, much of the Clinical Diagnostic skills that TJD dazzled us with was also learnt the same way.

A Seminal Paper by TJ Danaraj The earliest report of high rates of CAD in Asian Indians was based on 9,568 autopsies undertaken between 1950 and 1954 in Singapore. This study showed a 7-fold higher rate of coronary atherosclerosis in Asian Indians compared to Chinese.

This was the first paper published to report a >3-fold higher CAD incidence and mortality among Asian Indians than in Chinese, suggesting racial differences in the incidence of Coronary Heart Disease.
Danaraj TJ, Acker M, Danaraj W, Ong W, Yam T. Ethnic group differences in coronary heart disease in Singapore: an analysis of necropsy records. Am Heart J 1959;58:516-526. Takayasus Disease is also known as Danarajs Disease

Kianseng Ng I now catch a glimpse of this Giant of a man! Medical students need to know medical history, postings like this inspire younger ones to excel, all these are of course a prelude and a foreshadow of the TJ D Month in April 12.

Kianseng Ng LKJ, Ransome, both were my teachers...LKJ can hear an EDM a mile away! Ransome was the old fashion MRI with his tendon tapper that located Brain lesions with better-than MRI accuracy, and Chew Chin Hin as well, my, what a gallery of the greats and now where are these great clinicians, the only ones left is WYO in JB and NKW in Melbourne and after that no more....Ah, the good old fashion days! Chee Yong Chuan I really want to be like them in the future. World-class physician, not technician!

Construction of the faculty building began in July 1963 right through March 1967 when the first wards were opened culminating finally in the completion of the Paediatric, Maternity and Rehabilitation Units which became functional in March 1968. On 5th August 1968, the University Hospital was officially opened by the Malaysian King.

Picture taken in the Temple of Knowledge at UM called the Clinical Auditorium where T J Danaraj was its High Priest of Medicine. My Classmates and I are at the feet of The Master. Author is 5th from left in the front row. Caroline Wong Dr Wong, I see where you got your stick idea from, (note long stick on the examination bed!)

In the 4th and 5th year we stayed at the Clinical Students Hostel adjacent to the University Hospital. This was to ensure that students could go to the wards and theatres conveniently at night. We all had single rooms which was a luxury then but it was a small Spartan room almost monastic in design; a concrete cupboard, table, chair and a bed was all the little room had within. Danaraj made sure we were NOT Too Comfortable inside!

Lecture Hall2 at the Faculty of Medicine is a formidable Hall 3 stories tall. When I first walked in as a 1st year medical student, I went "WOW!" There were 3 Blackboards, and chalk and duster was still the means of teaching.

In the good old days, exam results were merely posted on the notice board by the office boy strolling casually out from the office after the External Examiners/Faculty meeting. Note the Clock; results typically were out just before 5pm. Everyone from the gardener to the amah knew how we did; this is a world away from the present times when results are send by email individually to the students and NO ONE knows how well or poorly they did UNLESS the student choses to tell us. Prof Khalid once exclaimed, "I am the Professor of Medicine and even I have no idea how my students fared!!" That's modern medical education!

Most times its the errant student who stands on the chair BUT occasionally even the great Professor of Medicine also stands on a chair!!! This is the Examination Hall where many stomachs churned, hearts went BOOM3 and legs weak!

The Beautifully designed Faculty of Medicine before modern progress changed it all!

This eBook is a Tribute to Prof TJ Danaraj from all the Teachers & students of the eLearning website, Aequanimitas. This Facebook website is the brainchild of Dr Ng Kian Seng, and is dedicated to the Teaching of the Art and Philosophy of Medicine, promoting its Ethics and Traditions.

In the days before Email and Facebook, Prof TJ Danaraj took great pain to write in long hand to us, encouraging and inspiring us to be better. I keep this letter with great pride and joy; in this era of SMS and Manglish, this culture is lost.

Ho Loon Shin In 1988, at the silver anniversary of the medical faculty, UM, TJ Danaraj he spoke about the vision and desire to help fellow human beings and to continue to be 'role models' in caring for people. In the speech he quoted passages of the Prayer of Maimonides (The Physicians Daily Prayer) "Inspire in me a love for my art and for thy creatures. Let no thirst for profit or seeking for renown or admiration take away from my calling Keep within me strength of body and of soul, ever ready, with cheerfulness, to help and succor rich and poor, good and bad, enemy as well as friend. In the sufferer let me see only the human being. If those should wish to improve and instruct me who are wiser than I, let my soul gladly follow their guidance; for vast is the scope of our Art. In all things let me be content, in all but the great science of my calling. Let the thought never arise that I have attained to enough knowledge, but vouchsafe to give me ever the strength and the eagerness to add to what I know. For Art is great, and the mind of man is ever growing. I can see so many of TJD's students doing that. Clap, clap, clap.

When his health failed, when Parkinson's ravished his physical body, Prof TJ Danaraj had to type his letters as his handwriting deteriorated. Both these letters mean a lot to me.

A Letter from Prof TJ Danaraj to you the Reader here!!! Dear ., Herewith a copy of the group photograph that we took on that memorable evening of the day when your results came out and you graduated. I have thought of all of you several times since then and in my minds eye I see you performing the many tasks and taking on the responsibilities of a young doctor in the complex system of a hospital. I hope this Faculty has given you the necessary independence and fortitude to face your problems and solve them. As your daily toil continues and the pressures around you increase, you will tend to lose sight of the heritage of Medicine and the principles and policies that we have talked about during your University course. Would you now and then stop looking down at the earth where you are working and look up to the stars because that is where your sights should be. Always aim high for yourself. Not just a hundred which is easy to attain but a million which you may not attain but certainly is worth trying for, like in the words of Browning: That low man seeks a little thing to do, sees it and does it: This high man, with a great thing to pursue, Dies ere he knows it. That low man goes on adding one to one, His hundreds soon hit: This high man, aiming at a million, Misses an unit. I look forward to seeing you at the Convocation this Friday when together with your other teachers, your parents and friends, we shall feel pleased and proud as you step up in your academic gown to receive your degrees. After the Convocation we shall take a group photograph of the whole class in the Postgraduate Museum. Please feel free to bring your parents and anyone else along so that they could be included as well. This will be the last time we shall be able to get all of you together so I do hope you will come soon after the Convocation for the photograph. With kind regards and all good wishes. Yours sincerely, T.J. Danaraj

Personal recognition was unimportant to him; the success of his students was his measure of success. He was Emeritus Professor of Medicine at the University of Malaya with the privilege to teach as long as he was able to, for with that honour is a Lifelong Professorship; and he indeed taught until his body was too frail to allow him to continue.

Money was collected for the TJ Danaraj Memorial Fund after his passing; this was donated to the Academy of Medicine for the Academy building. The main auditorium at the Academy of Medicine Building is now named after Prof TJ Danaraj in his memory

Prof TJDanaraj spent a lifetime teaching Medical students to be good doctors. He not only taught the Clinical Art of Diagnosis, he taught the Philosophy on how to be a good doctor; to this very day his students emulate his methods and philosophy when they teach their students in turn.

TJ Danarajs lifelong collection of Slides

4 solid metal safes full of slides were given to me by Prof HO Wong, Prof TJ Danaraj's widow. A boxful of glass slides on Prof Danaraj's MD thesis research on Tropical Pulmonary Eosinophilia done at the Singapore General Hospital has been given to the SGH museum for display and safekeeping. These slides bear witness to a lifetime's dedication to the Art of Medicine and education. Most of the pictures in this eBook are from his collection.

KianSeng Ng April 2012 has been designated Professor TJ Danaraj Month on Aequanimitas. And the Teacher for this inaugural event will be Assoc. Professor Wong Yin Onn. For the month of April, Assoc. Professor Wong will be the only one posting and teaching. We want to make this event worthy of honoring Prof TJ Danaraj as the teacher extraordinary, the great human being, the phenomenal Oslerian Clinician, the one who pointed the way and who blazed the path...

From the stroke of midnight, I am commissioned to post a lesson a day; lessons inspired by the great medical educationalist Emeritus Professor TJ Danaraj. It is Not an easy task for although I possess thousands of slides taken by him, to reflect accurately the thoughts of a Man into words without distortion or glorification demands objectivity that is not clouded by the passage of time. I also have a file of articles collected by him regarding medical education, articles ranging from the introduction of OSCEs in the west in the late 70s to specialisation fragmenting the practise of medicine feared and predicted by American doctors back in the 70s. From all these material and my own memory, I will try to organise a Path; a path that can be walked by the medical students with me over the next one month to learn together this ancient and fine art. To my Monash students who inspire and frustrate me equally, let us learn daily. To the IMU students whom I have not met before, we will be digital friends on this walk. To the UM students, you are the descendants of a great legacy never to be forgotten. You must strive to achieve to greatness as envisioned by your direct medical ancestor. To all the other students, I hope I can inspire you as my Teachers inspired me. To all my colleagues, please correct me where I stumble and forgive me where I fail. Dr Wong YO

Dear Friends, I am writing this on the eve of a momentous event, The Emeritus Professor TJ Danaraj Month on AeQ, Aequanimitas. April is designated to honor a Pioneer, the Legendary Teacher Extraordinaire, the Oslerian Physician. And one of TJDs protg at UM was the young Wong Yin Onn and today Yin Onn is Assoc. Professor in Internal Medicine at the Clinical School Monash University in JB. Assoc. Professor Wong Yin Onn is the Honored Teacher for this inaugural TJ D Month. For the whole month of April, the whole virtual space on AeQ is his border-less lecture hall and we are his rapt audience. I dont know what Prof Wong will be doing but when he Posts, I will train my eyes on the writing on the Wall, I will try to decipher the hieroglyphics of the patients story of his malady, I will try to take the jig saw pieces of his dis-ease and create a holistic image. Above all, I will incline my ears to listen to the heart beat of the healer and the heart cries of the one asking for healing. Your comment will be worth its weight in diamond! When you stumble over a difficulty, ask to be enlightened. Medical students, dont ever believe that your question may appear foolish. The only foolish question is the question unasked. You may want to Post, you are not trespassing when you enlarge, elaborate on a point Prof WYO is emphasizing, this is not a didactic session, it is a dialogue between a master and his students. And the pearls will surely fall and then over flow your treasure chest. And in the process of learning and sharing we all become what Emeritus Professor TJD wanted his students to become, the-best-we-can-be-physicians bringing the balm of healing to a suffering world and bringing along also a mega dose of our love! May April Be Your Spring Delight As It Is Already Mine,

Kianseng 31 March 2012

1. The Start of a Diagnostic Process

The History

History taking is the foundation of the Clinical Art and the heart of the diagnostic process in Medicine. The medical history is the grand centerpiece, the big picture that provides a panoramic overview of the patient's entire illness - how it originated, how it grew and developed, and how it is at present. It is primarily the medical history that tells the doctor which specific signs to look for, and what other subsequent investigations are needed to obtain the information he seeks. All other methods of diagnosis can be seen as auxiliaries to the medical history, which is the principal or core diagnostic method. Prof TJ Danaraj demanded a highly detailed history from his students, and a sloppy history will be met with a severe reprimand. Prof KK Toh recalled how a student who did a particularly poor job will be exiled from the Bedside teaching until he has personally apologised to the teacher and each student in the group, for he has wasted the precious learning time of the entire group and himself; and insulted the patient. TJD will typically draw a long line across the blackboard with his chalk; the student will be asked as to describe in detail what happened on the Day1 when the first symptom appeared and then Day2 etc. Then what brought the patient to hospital, that precipitating event which made the patient seek admission, and subsequently what happened Day1 of admission, etc all the way to the present day. We must pay close attention to the time course of the symptoms. How has symptom complex changed over time? The natural history of the illness, its progression is of utmost importance in diagnosis. Today I am saddened that most students take lightly this "History of Present Illness (HPI)". In this era of instant coffee and maggi mee, the students compressed all this HPI into a few brief statements, losing much that it tells us. Always remember that The History is the Patient telling us the Diagnosis!

I tell my students to at the very least think of 5 common conditions that can explain for the Chief Complaint and HPI. For eg if the Complaint has been Chest Pain, then at the very bare minimum, think of 5 Common or Important conditions that can explain the HPI. The Common include Ischaemic Heart Disease, the NOT TO BE MISSED include Acute Myocardial infarct, Aortic Dissection, Pulmonary Embolism, and Pneumothorax. The long list of others from GERD to Oesophageal spasm to Zoster is at the back of our minds. With this we review the Systems and LOOK at each system one by one comprehensively for possible symptoms and disorders that may not be spoken of by the patient in the HPI. In the HPI the patient volunteers his data, in the systemic review we enquire! The focused review of systems bring out information that supports a certain diagnosis or helps gauge the severity of the disorder, or exclude the likelihood of a pathology. Today, the systemic review often consists of mindlessly repeating a few lines that goes.... "Patient does not smoke, drink or take drugs of abuse. He has no sexual indiscretion". This is one extreme. Any book on Clinical Methods will have a list of symptoms that comprised the definitive "Review of Systems." Some students actually memorized this list. This is the other extreme! But the Mindless recitation of this list is rather stupid: "I know that you are having chest pain, but I need to know if you have ever had an extra marital affair."

The Diagnostic process is time honoured, refined by the passage of time, a distillate of Medical wisdom. History taking and Physical examination remains the pillars of sound practise, no matter how the machines have advanced. The human touch, both the spoken word and the touch of the hand is as important as the pharmacopeia. Even if the superb diagnostician may not need it, the suffering man does. he consults a doctor, not a machine, he seeks help, not the beeps of computers. On complicated machines we now rely, and use ear, tongue, eye and hand far too little. We scan and sound everything, and await with pious resignation the decree of the computer. Clinical methods we tend to damn, and the doctor now becomes a stranger to the patient. The modern doctor is at ease with ECHOs and Scans, but he forgets the symptoms and signs of diseases, the patient now a nameless collection of body parts to be referred to organ specialists, where once on a clinician's skills the matter rests. But we have yet a machine that can measure Human pain and distress, nor a computer that can counsel and relieve. What ails a man from the symptoms and signs, a clinician can tell, at the bedside he truly shines, his conversation reassures the patient that he is the most important suffering being to this doctor, his touch a soothing balm to the aching body.
TJD warned us that we are Doctors, NOT Technicians; and unless we continue to act as Human Doctors healing the sick Human being, the Technician will take over! The History is the Patient telling us the Diagnosis, The Physical Examination is the Body telling us the Diagnosis.

Both must be Complete. The History is in a language both the Doctor and Patient understands, The Physical Examination is in a language only the welltrained doctor understands.
The Mathematics of Diagnosis is the Mathematics of PROBABILITY; what is the probability of disease A causing the symptom complex of this patient? In the OSCE method of evaluation of our medical students, the student is asked at the end of his history taking... "What is your Provisional Diagnosis and Differential Diagnosis at the end of this interview?" Effectively we are asking, what is the Probability of the diseases causing this illness from the highest to the Lowest! Some symptoms provide us with valuable clues to the diagnosis, for eg Fever, Chills, and Rigors! Please give me 5 important causes. One of my students based overseas saw a patient come in with Fever, Chills and rigors, abdominal discomfort and unilateral flank pain. From the history, he was able to quite accurately localise the source of infection that took the attending doctors a CT Chest and Abdomen to diagnose! The history and physical examination should give us a Working Diagnosis based on which we order the appropriate Investigations. It should not be that we do the Investigations to give us the working diagnosis! The medical history and dialogue between the patient and his doctor is the heart of the doctor - patient relationship. It is here that the doctor establishes a rapport with his patient, and communicates to him/her his sincere caring and commitment to their recovery. This caring in itself has great therapeutic value. The cold impersonality of modern medicine is anti-therapeutic, and de-humanising. Kianseng Ng Remember this clinical gem, No 1,

"The History is the Patient telling us the Diagnosis, The Physical Examination is the Body telling us the Diagnosis." "By the time you finished taking the history you should already (More or less) know the diagnosis." It is not just the long narrative but the manner in which it is told and the whole body language of the one telling the story, the facial expression, the hands, the lower limbs, the words especially the adjectives used...I tell medical students that often the "symptoms" are the only "positive signs" and the accompanying physical examination may very well be negative or NAD (No Abnormality detected). Kong Wah Ng Make sure that NAD does not stand for "Nothing Actually Done"! Mario Yoong Well done! Prof Wong. Back to basic and how really important this is especially to us, the GPs. We do not have the good fortune of machines and computers to help us, so in 90+% of the time we have to depend on history, medical examination and basic equipment to help us. Occasionally we get to see a few exotic diseases but they are far and few. Medicine is not about rare and exotic diseases. The Golden Standard of medical practice is still the basic History, Inspection, Palpation, Percussion and Auscultation and let us all not lose sight of this. Punna Wong Please do not Forget LOBAR pneumonia, Ascending Cholangitis, Acute Pyelonephritis, Septicaemia, Abscesses and Malaria when you see a patient presenting with Fever, chills and Rigors. Any cause of Sepsis can give rise to this symptom. There are of course other causes from Dengue to Scrub Typhus but never forget the common ones that are treatable when picked up early. And always remember that a NORMAL WBC means NOTHING! One could be severely septic and have "Normal" counts!

2. The Physical Examination The Physical Examination begins the instant the patient walks into the room, or the students start presenting the history and I am standing at the foot of the bed observing the patient. If there is ONE thing that I learnt from TJD, it is to OBSERVE very carefully and intently. Many a diagnosis can be made at first contact; the patient limping into the room with gout, the patient shuffling in with Parkinson's or the Hyperthyroid patient with starry eyes and goitre. Once a classmate presented to TJD a patient who came in the night before with seizures. TJD walked into the room, looked at the patient and walked around her and then BOOMED "What is the diagnosis???" As medical students we were shell shocked, nary a word had been said and the great man demands a diagnosis! TJD then explained and demonstrated to us the signs that told the diagnosis. Remember that in the Physical Examination, the Body is TELLING you the Diagnosis! What is the Body telling you here?

What is the cause of this patient's seizure?


Arif Rahim This is uremic frost of severe uremia, which would also cause seizures (uremic encephalopathy). This would warrant a BUSE/creatinine to confirm the uremia, keeping in mind to also investigate the cause (both ARF and CRF). Other signs such as nail changes, pruritis, dehydration, uremic fetor, asterixis may also be present. 3. You Look But you DO NOT See! As the patient walks into your consultation room, you notice that she has to pause and breathe; every breath appears a valuable asset to be treasured and she gasps with effort but with little relief. She squats down and for a while looked less distressed. You have not even begun your formal routine examination But astute eyes will have picked up much information.

Pls describe the Physical Signs and the patho-physiology behind the patient's behaviour. Prof Dr. Joseph Bell, a brilliant Diagnostician, was the man who inspired the character of Sherlock Holmes; Conan Doyle put his many qualities into the famous detective.

He said "In teaching the treatment of disease and accident, all careful teachers have first to show the student how to recognize accurately the case. The recognition depends in great measure on the accurate and rapid appreciation of small points in which the diseased differs from the healthy state. In fact, the student must be taught to observe. To interest him in this kind of work we teachers find it useful to show the student how much a trained use of the observation can discover in ordinary matters such as the previous history, nationality and occupation of a patient." In the education of a student of medicine, he MUST be taught how to observe; the brilliant student who enters medical school is already gifted with eyes, memory, and imagination. Now his Eyes and ears must be trained to see and hear the language of diseases, his voluminous memory used to record at once much theory and recall it at pleasure. All his 6 senses, the 5 sense bases plus his Mind must be utilised to his advantage to weave together a diagnosis that explains the chain of symptoms unraveling all the tangled clues. Such are the tools of a successful clinician.

Anver Sethwala This picture demonstrates a young thin girl with clubbing of the fingers and toes. She is having a hypercyanotic or 'tet' spell of tetralogy of Fallot. When she squats, this increases systemic vascular resistance and allows for a temporary reversal of the shunt. It increases pressure on the left side of the heart, decreasing the right to left shunt thus decreasing the amount of deoxygenated blood entering the systemic circulation. Lau Ron Hsien Comprising of 4 distinct conditions 1. right ventricular outflow tract obstruction 2. right ventricular hypertrophy 3. large VSD 4. over-riding aorta arising slightly superior to the large VSD. Depending on the degree of the right ventricular outflow tract obstruction, a patient may appear pink (if only mild obstruction) or cyanosed (if severe obstruction).

4. The Hands

The hands are beautiful organs of expression! It talks of Hope, Love, Joy, Pain and Sorrow. TJD put Great emphasis on the detail inspection of the entire patient. He will look at the hands mindfully and unravel the many stories carved there. We shake hands and proceed with its inspection, going in a circle like manner from hands to elbows to face, neck, chest all the way down to the feet.

What does this hand tell you? Think of the common and the important first and let us start with 10 causes. This sign can be very subtle; the student must learn to pick up early changes, pls tell me what is the earliest change? But such signs must never be seen in isolation! What does this sign in the context of the History and other Physical Signs tell us? That is far more important!

This patient with the same finding on his fingers is asked to stretch out both his hands straight, now what is the likely story behind this finding? Lau Ron Hsien Drumstick fingers, a sign seen in patients with advanced digital clubbing. The first sign is the increased bogginess of the nailbed. One of the clinical signs to elicit would be to check if Schamroth Sign is present. The sign is positive if the diamond shaped "window" is loss upon opposition of the fingers nailbed to nailbed. Causes can be classified into... Cardiac - Cyanotic heart diseases, Subacute Bacterial Endocarditis Respiratory - Neoplastic lesions, Bronchiectasis, Cystic fibrosis, Pulmonary fibrosis GIT - Inflammatory Bowel Diseases (Crohn's and Ulcerative colitis), Primary sclerosing cholangitis, primary biliary cirrhosis Endocrine - Thyroid acropachy Others - Empyemas and abscesses, and last but not least, congenital There is most likely compression of the brachial plexus by a pancoast tumour. Look for signs of Horner's Syndrome (ptosis, myosis, anhydrosis, enophthalmos). And as with all kinds of primary pulmonary carcinomas, keep a sharp eye for paraneoplastic syndromes, of which, the presence of Cushing's Syndrome, SIADH, or Lambert-Eaton Myasthenic Syndrome would suggest that the tumour is Small Cell in nature.

5. The Hands that Taught you

The Hippocratic oath is specific regarding the Bond between the student and the Hands that taught him. TO RECKON him who taught me this Art equally dear to me as my parents, to share my substance with him, and relieve his necessities if required; to look up his offspring in the same footing as my own brothers, and to teach them this art, if they shall wish to learn it, without fee or stipulation.

Now retired teacher comes to see you complaining of Tiredness and Dyspnoea on exertion. What does Prof have? What Investigations are urgently needed?

Lee Chiang Sheng It is koilonychia/ spoon shaped nails indicating iron deficiency anemia. The symptoms of tiredness and dyspnea on exertion signifies symptomatic anemia. Other symptoms to look out for are palpitation and syncopal episodes. The main ddx in an elderly prof who presents with iron deficiency anemia is GI malignancy. Other causes would be peptic ulcer disease, chronic diseases, diet. History looking out for recent change in bowel habits (constipation/loose stools), bleeding PR, abdominal pain/distension, LOW/LOA would be very important. Besides, history of NSAIDs, aspirin, corticosteroid consumption, family history of GI malignancy would be needed. Having to know the relevant history from the patient, it's time for the body to guide the diagnosis. Physical examination for vital signs (low BP, tachycardia, tachypnea), signs of anemia (conjunctival pallor, pallor of palmar creases), Cardiovascular and respiratory examination, abdominal examination for any palpable mass and most importantly a PR examination! A simple FBE would guide the diagnosis as to what might be the cause. Hypochromic microcytic would suggest iron deficiency anemia, Thalassemia. Iron studies would be needed to confirm IDA. I would advise Prof to undergo colonoscopy to look out for any malignancy Anver Sethwala Picture demonstrates Koilonychia suggestive of iron deficiency anemia. Milder forms are better felt than seen, and should be present in more than one limb in order to rule out other causes of nail dystrophy. IRON DEFICIENCY ANEMIA IN AN ELDERLY IS COLON CANCER UNTIL PROVEN OTHERWISE.

6. More Hands for you to handle

Pls give me 5 other Major signs that you will expect to find on General Inspection that will confirm your diagnosis. Lee Chiang Sheng Short stubby hands with clinodactylyl in the 1st picture along with sandal gap deformity between the first and 2nd toe for the 2nd picture pointing towards Downs syndrome. Other signs I would look for are single palmar crease, typical facies (medial epicanthic fold, low lying set of ears, upslanting palpebral fissure), protruding tongue (due to hypotonia), Brushfield spots/cataract. Hearing difficulties, congenital heart defects (AVSD, PDA, VSD are common), hypothyroidism, diabetes, GI complications (duodenal atresia) are among the other complications to look out for. Chee CheongFun The pathogenesis is caused by 1] 95% of it by meiotic nondisjunction of homologous chromosomes (usually associated with advanced maternal age), 2] robertsonian translocation and 3] down mosaicism with no maternal association. Has high risk for ALL as well as Alzheimer by the age of 35. During pregnancy, the quad screening are 1] decrease in AFP and estriol and 2] increase in B-hCG and inhibin A. Make it clear to the ladies who wants to get pregnant at an older age that statistics show that ratio is at 1:1500 for woman less than 25 y/o and 1:23 for woman more than 45 y/o.

Another Lo Si from Kluang with its Lovely food came to see you. What is your clinical diagnosis? What will you do for the poor man?

You roll up Lo Si's sleeves and what do you see!!

Tophaceous gout. A history of acute gouty arthritis with painful swollen joints would suggest the diagnosis. Other risk factors are diet high in purine (RED MEAT, SEAFOOD), alcohol and medications (thiazide diuretics). Obvious wasting of the interossei muscle and tophi over 2nd metacarpophalangeal joint of both hands and left elbow is seen. Hyperuricemia is also associated with increased risk of CVD events, uric acid nephropathy, renal stones. I would educate Lo Si on lifestyle changes especially his diet, alcohol consumption and other comorbid conditions.

Tommy Lai Allopurinol treatment only indicated if patient has one of the following, recurrent gouty attack for >3 times in a year, serum UA of more than 0.77mmol/L and evidence of complications of gout (renal impairment & stones, gouty tophi).

7. Hands sadly Destroyed

This is one of the tragedies that cripple a person. The fingers are now flailed and functionless, bones and joints destroyed. What is the Diagnosis? Ian Goh with the overlying skin changes this is arthritis mutilans seen in psoriatic patients. Other features seen are onycholysis, telescoping of digits & dactylitis.

This could have been prevented. If you are to take an XR now, what will you see?

In Chronic Tophaceous gout) a) Joint findings * Erosion of joint margins with sclerosis * Cartilage destruction late in course of disease b) Bone findings * "Punched-out" lytic bone lesion sclerosis of margin * "Mouse / rat bite" from erosion of long-standing soft-tissue tophus * "Overhanging margin" (40%) * Ischemic necrosis of femoral / humeral heads * Bone infarction

Hu Mung Chee Chronic Tophaceous Gout. Clinically, both gouty arthritis and septic arthritis can present with fever and leucocytosis and this makes differential diagnosis difficult. If this is the first attack and the patient has no past history of gout, I will perform arthrocentesis to differentiate this two. Ramachandran Arumugam There is normouricaemic gout. And the sine qua non of acute gout is the demonstration of negatively birefringent monosodium urate crystals.

Can we differentiate Gout from Rheumatoid Arthritis clinically?

What could have caused this? Describe the anatomy behind this finding

Lau Ron Hsien Volkmann's Ischaemic Contracture results from the ischaemia and possible necrosis of the flexor group of muscles of the forearm, causing the muscle fibres to be fibrosed and contracted. Most commonly caused by a supracondylar fracture of the humerus, but can also be caused by the improper uses of a tourniquet or plaster cast. Hu Mung Chee This is "claw hand deformity" (not partial claw hand deformity). This deformity may due to medial epicondyle fracture or Guyon Canal entrapment, results in paresis of the ulnar and median nerves, in which there is extension of the metacarpophalangeal joints and flexion of the interphalangeal joints, with associated flattening of the hand due to musculotendinous imbalance. In ulnar nerve injury, the hand appears as a Benediction Sign, Bishops hand or partial claw hand, whereas Claw hand deformity is typically the result of a double injury to the median and ulnar nerves.

Many conditions can give rise to this. Pls name them so that we can learn to prevent this

Dry gangrene of the fingers. If the patient is a smoker, the possibility is Thromboangitis Obliterans. Thromboembolic events can also result in digital gangrene. Disseminated Intravascular Coagulation, Heparin Induced Thrombocytopenia and Diabetes Mellitus can also present similarly. Severe long standing Reynaud's Phenomenon, CREST Syndrome can present with ulcerations of the fingertips, which can progress onto ischaemia and gangrene of the finger.

One too many fingers, what are its associations? Caroline Wong Polydactyly can occur on its own but also associated with many serious congenital anomalies and syndromes such as bardet-biedl syndrome, VACTERL association, Rubstein Taybl syndrome..

Polydactyly and syndactyly are usually isolated conditions, ie the child will have no other birth defects; however, both are also found in many complex and sometimes lethal syndromes. There are more than 100 different genetic syndromes that have polydactyly as one of their findings. If the extra digit is located on the side of the hand by the thumb or on the side of the foot by the big toe, then a person is diagnosed with preaxial polydactyly. Postaxial polydactyly occurs when the extra digit is located on the side of the hand or foot by the fifth digit (the pinky or small toe), and central polydactyly occurs when the extra digit is located in between the thumb and pinky or between the big and little toes. Preaxial polydactyly is the most common form of polydactyly in individuals of Asian ancestry. Postaxial polydactyly or an extra pinky finger, is the second most common malformation of the hand, occurring in approximately one in 3,000 births. Most often these conditions are caused by genetic factors.

8. Protect your MBBS well!

1. TJD insisted that we MUST examine our patients comprehensively NO MATTER what the complaint is. Every part of the Human body is related to every other part. I am reminded of my Consultants advise when I reported to her department as a young doctor "You had worked very hard to get your MBBS, now make sure you protect it well!"

Look at the pair of hands above. The "focused' examination so prevalent among my charges vs the Comprehensive Physical Examination we demand would have made a HUGE difference to this patient. WE MUST EXAMINE THE WHOLE PATIENT, NEVER A PART OR AN ORGAN! IF THERE IS ONE THING YOU MY STUDENTS MUST LEARN FROM ME, LET IT BE THIS!

2. This is the patient's buttock, would it not be to the eternal shame of the doctor if he had missed this because he merely did a "Focused" examination? What is your diagnosis? What must you do now to manage this patient AND his family?

Hu Mung Chee The diagnosis is familial hypercholesterolemia. Figure1 shows tendon xanthomas over the extensor tendons of the hands. Figure 2 shows multiple tuberous xanthomas over buttock. Tuberous xanthomas are nodular, firm, painless subcutaneous swellings that develop in those areas that are subjected to repeated trauma such as elbows, knees, and buttocks. They may arise as single or multiple and vary in size from pea sized to lemon/mango sized. Sometimes they may coalesce to form large tumorous swelling. Tuberous xanthomas are usually associated with raised low density lipoprotein levels. This patient may be misdiagnosed if the student does not examine the patient thoroughly. Punna Wong When do we suspect Familial HyperCholesterolaemia? 1. When the Cholesterol level is >7.5mmol/l without secondary causes like Hypothyroidism, Nephrotic Syndrome, DM or liver diseases 2. Family history of IHD <50 yrs old in 1st or 2nd degree relatives 3. Cholesterol level of >7.5mmol/l in at least 1 1st degree relative 4. Presence of Tendon Xanthomas and Arcus juvenilis will increase the probability.

Kong Wah Ng Primary dyslipidaemias are associated with overproduction and/or impaired removal of lipoproteins. The latter defect is caused by an abnormality either in the lipoprotein itself (eg defective apolipoprotein B-100 ) or in the lipoprotein receptor. No need for sophisticated tests, just request serum total cholesterol, triglycerides, HDLC and LDLC. The incidence of dyslipidaemia is highest (up to 85%) in patients with premature coronary heart disease defined as occurring before 55 to 60 years in men and before 65 years in women. The three most common types of primary dyslipidaemia are autosomal dominant hypercholesterolaemia (ADH), familial hyperchoesterolaemia (FH) and combined hyperlipidaemia.

ADH is a clinical disorder characterized by elevated plasma LDLcholesterol (due to reduced LDL clearance) and premature atherosclerosis. FH is the most common of these and is caused by defects in the LDL receptor gene. Familial defective apolipoprotein B-100 is less common and leads to impaired binding of LDL particles to the LDL receptor. Some of the excess LDLC is deposited in the arteries as atheroma and in the tendons and skin as xanthomata and xanthelasma. Homozygous FH is rare (1 in 250,000 births) while heterozygotic FH is more common, affecting 1 in 500 people in USA. Tendon xanthomata and premature atherosclerosis can also occur in two rare disorders not involving LDL metabolism: sitosterolaemia an autosomal recessive disorder associated with hyperabsorption of cholesterol and plant sterols from the intestine, and cerebrotendinous xanthomatosis which is characterised by a block in bile acid synthesis. It is associated with prominent neurologic abnormalities and usually normal serum cholesterol. Eruptive xanthomata, tuberoeruptive xanthomata and palmar xanthoma are observed in familial cases of hypertriglyceridaemia. The three common conditions are fasting chylomicronaemia (type V), familial hypertriglyceridaemia (type IV) and familial combined hyperlipidaemia. A characteristic of hypertriglyceridaemia is the presence of milky plasma (colour of strawberry cream) and lipaemic retinalis. Note that if serum triglycerides >4.5 mmol/l, the calculation of LDLC from the Friedewald equation is not accurate. Friedewald equation: LDLC = Total cholesterol (HDLC + TG/5). The rationale for the management of primary dyslipidaemia is the prevention of premature coronary artery disease. Long term studies have shown that for every 10% reduction in serum cholesterol, coronary heart disease mortality can be reduced by 15% and total mortality risk by 11%. This involves (1) Lifestyle modification (2) Pharmacological therapy (3) Addressing other risk factors for cardiovascular disease

Success or failure depends on many factors eg motivation, age, socio-economic status, income, cost etc and recognition that these are long-term measures not something that can be achieved in a couple of weeks or months Lifestyle modification: This is the simplest and cheapest part of the management program but is often the most frustrating because goals are often not met. Weight loss in overweight patients. Present patients with an achievable target, eg 7% to 10% weight loss over 6 months and then maintained at the lower level. Give them a detailed program of exercise eg walking 30-40 mins each day. You may soon accumulate a database of excuses as to why they cannot exercise (too hot, too cold, raining, too much sun, too busy etc), but persevere anyway. Tip: - see them at short regular intervals to monitor progress. Diet: Get into their heads. Take a detailed history of what they eat everyday and how much. Then speak to a dietitian to get appropriate advice with regards to Chinese vs Muslim vs Indian diets. Families have to get involved more likely to succeed if everyone in the family has a similar diet Pharmacological therapy: Statins, probably in high doses usually required. In certain cases, patients can be as young as 8 years old. Be aware of adverse effects eg myositis. If only partially successful in lowering LDLC, may need to add alternatives such as Ezetimide (can achieve further reduction in LDLC but no data showing improved clinical outcomes cf high dose statins alone), nicotinic acid, questran (bile acid sequestrant). Compliance with other agents is often an issue. Patients with severe hypertriglyceridaemia also faces risk of acute pancreatitis. Insulin infusion often used successfully to acutely lower hypertriglyceridaemia in an emergency. Fibrates lower triglyceride levels eg gemfibrizol, fenofibrate (myositis also a problem esp when combined with statins). Nicotinic acid can increase serum glucose levels.

9. A Flowery Rash

Rashes on the Palms are always a RED flag. Give me a few Impt conditions that can present this way.

What is the most likely diagnosis? How will you confirm the Diagnosis? How will you manage this patient? Anver Sethwala Typical copper coloured macular rash on palms and soles of secondary syphilis. Examine for Lymphadenopathy (esp epitrochlear), secondary mucus patches in oral cavity or genital area, condylomata lata including around the perianal area, eye signs (if they have ANY eye signs treat as for neurosyphilis), as well as look for other STDs eg: oral candidiasis signifying immunosuppresion of HIV. VDRL and TPHA/TPPA (note VDRL maybe negative in late latent syphilis). Treat with IM penicillin stat. For secondary syphilis contact tracing up to 2 years is recommended. Partners get treated irrespective of symptoms or serology

Punna Wong Causes of Rash on Palms and Feet 1. Hand-Foot-Mouth disease 2. Atopic dermatitis 3. Contact dermatitis 4. Erythema multiforme 5. Syphilis, secondary 6. Rocky Mountain Spotted Fever 7. Tinea manuum 8. Kawasaki 9. Measles, Rubella 10. Psoriasis 11. meningococcemia 12. Scabies 13. Palmar and Plantar Keratoderma 10. What could have caused this?

William Liong The lady has unilateral left upper limb lymphedema and tethering of ipsilateral breast, asymmetric nipple level and size of breast. She has Left breast Ca with Left axillary LN involvement

11. Our Journey reaches the axilla

This patient works at an Oil Palm plantation. He had high fever and chills, drenching sweats, severe headache and retro-orbital pain, and subsequently developed seizures. A medical student found this. What is the likely diagnosis? Anver Sethwala An eschar of scrub typhus caused by the intracellular parasite Orientia tsutsugamushi, and transmitted by the chigger mite. Hu Mung Chee Picture shows "Tache noire" or "eschar" over right axilla. Seeing this physical sign can help make the diagnosis easily, but we miss this if we do not do a detail physical examination.

And this is what I mean when I say that a doctor who does NOT follow the Golden Precept of Examining Meticulously will miss. The diagnosis is obvious if the doctor pulls down the pants and look! I keep telling my students "No shy2 nonsense! If you want to be a doctor, you have to see, touch and feel no matter where and who!"

Kianseng Ng The eschar is pathognomonic but often missed. In adults, the eschar is often truncal, whereas children may have lesions in the perineum. Multiple eschars may be present. Some patients may have CNS involvement with tremors, nervousness, slurred speech, nuchal rigidity, or deafness, seizures during the second week of the disease. It is said that if acute hearing loss is present it strongly suggests scrub typhus. Eschar on Trunk

Infection occurs when rickettsiae are introduced through the skin by the bite of a mite, or by the rubbing of infectious feces into the skin. A necrotic lesion (escar) is common. Several days after the appearance of the ulcer, fever, headache, sweating, blood-shot eyes, swollen lymph nodes, rash, lung infection, vomiting and diarrhea occurs. Multi-organ failure may result. This may be manifested most commonly by confusion or stupor, difficulty breathing, kidney and liver failure, as well as shock. Heart problems may occur. Clotting problems may cause gangrene. Kianseng Ng Some evidence of pulmonary involvement such as cough, tachypnea, or pulmonary infiltrates is often present. Cardiac involvement is often minor and rare; however, cases of fatal myocarditis have been reported. Infection with O tsutsugamushi may cause a relative bradycardia, which, when combined with rash, may raise concern for typhoid fever. Scrub typhus may rarely cause acute renal failure, shock, and disseminated intravascular coagulation (DIC).

12. We now reach the Face!

This series of pictures show the transformation of a young man. Pls describe the changes, the expected signs and complications. How will you investigate this patient? Chee Yong Nian There is coarsening of face and widening of his nose. In pic 3, the most prominent feature is the large, spade-like hands. All these signs are highly suggestive of acromegaly. Other expected signs are like tall stature, increased size of shoes, macroglossia, widely spaced teeth, loss of axillary/pubic hair, profuse sweating, skin tags, proximal muscle weakness and sometimes bitemporal hemianopia. Complications would be impaired glucose tolerance in most, heart problems such as left ventricular hypertrophy and arrthymia, increasing visual field defect. In order to investigate this patient, blood glucose level and serum growth hormone and IGF-1 should be measured. If basal serum growth hormone is >0.4g/L and IGF-1 level is high, there is a need for oral glucose tolerance test (OGTT). To confirm the diagnosis, MRI of brain can be done to check for pituitary tumor.

Yeoh Seng Hong Complications of Acromegaly: 1.Cardiac complications (Left ventricular hypertrophy, cardiomyopathy, arrhythmia, valvular heart disease, and ischaemic heart disease) 2.HTN 3.Sleep apnoea 4.Osteoarticular complications (articular cartilage hypertrophy, joint laxity, bone enlargement) 5.Impaired glucose tolerance and diabetes 6.Pre-cancerous colon polyps 7.carpal tunnel syndrome 8.Hypopituitarism (compression of the normal pituitary gland by the adenoma or secondary to treatment (surgery and/or radiotherapy))

Kong Wah Ng (1) what is the role of oral GTT in the diagnosis of acromegaly and (2) why do some acromegalics complain of galactorrhoea?

Yu Yen GH is suppressed during GTT. if GH > 1ug/L with clinical features, its suggestive of acromegaly (but can have false positive eg. in ppl with DM, renal/hepatic failure).

Ying Jie Liow Patients with acromegaly can present with galactorrhea because of the "stalk effect". GH-secreting pituitary adenoma can compress the pituitary stalk, and this interferes with delivery of Dopamine, also known as PIH (prolactin inhibiting hormone) from the hypothalamus to the lactotrophs in the pituitary. This causes elevations in PRL and thus patients present with galactorrhea. Kong Wah Ng If this acromegalic works as a lorry driver, do you think it would be safe for him to continue to work in his occupation before treatment? Eng Kian Ng Considering the fact that there is a large somatotrophic macroadenoma that potentially may compress the optic chiasma causing visual field defects (eg. bitemporal hemianopia),

one should not allow a lorry driver (or for that matter, anyone who works as a driver of large vehicles) to work without some form of visual field assessment. Tatsuki Abe the right thing to do is to report to the authorities after telling the patient that we will. The more practical question is will ANY doctors do that? The same goes with patients who drive a masstransport vehicle (eg. pilots) with psychiatric illnesses like schizophrenia. It is hard, but we need to do the right thing. Kong Wah Ng The consensus criteria for the diagnosis of acromegaly (JCEM 2000:85;526) state that acromegaly is excluded if randomly measured GH is <0.4 ng/ml and IGF-1 is within the age- and gender-matched reference range. If either of these is not achieved, either a 75g OGTT should be performed with GH and glucose being measured every 30 mins for 2 hrs or 24 hr sampling for GH. A GH nadir of <1 ng/ml during an OGTT or a mean 24 hr GH of <2.5 ng/ml excludes acromegaly. Both GH and IGF-1 should be measured, recognizing that there will occasionally be patients in whom the results will be discrepant. Please note that in patients with overt diabetes mellitus, the OGTT should not be performed to diagnose acromegaly. 30% acromegalics present with galactorrhoea. This could be due to one or both of two factors - stalk compression by the macro adenoma and/or co-secretion of prolactin from lactotrophs. These are the patients who can potentially respond to cabergoline as a second line therapy. 13. Punna Wong Today I share on Aequanimitas one of the MOST IMPT lessons I learnt as a medical student from TJD. He taught us that in making any diagnosis, there MUST be 4 components to it. 1. 2. 3. 4. Anatomical Diagnosis - where is the site of the disease? Pathological Diagnosis - what is the underlying pathology? Aetiological Diagnosis - what is the cause? Functional Diagnosis - what is the functional state of the patient now?

A student who answers (like so many of mine now) "Pneumonia sir" will likely get his leg whacked by the tendon hammer! (Sadly, I cannot use this 'methodology' in teaching nowadays, but it will definitely laser the fact into the students brainstems!) It is NOT just Pneumonia BUT "Right Lower Lobe Pneumonia, probably Streptococcal, with patient not in Respiratory Failure" or "Congestive Heart Failure from Chronic Rheumatic Heart Disease with Mixed Mitral valve disease and patient in NYHA Class3" or "End stage Renal failure from Chronic Glomerulonephritis with acidosis and massive fluid overload precipitated by sepsis" I hope medical students learn this well.

Ming Lee Chin I remember this so clearly. It was one of the very first lessons you gave us! Thanks Dr. (: Kianseng Ng Monash students last year at the CXR workshop would have learnt from me and Prof Wong the 5th Diagnosis, "Patient and wife came in a taxi from Kluang, they do not own a car!" I hope you students who come from rich backgrounds realise the implications...and respond with big hearts! Carry this "5th" Diagnosis with you as you minister to all your patients! When you make the first 4 aspects of the diagnosis, you are already a excellent doctor, but when you make the fifth aspect of diagnosis, you are an excellent, compassionate doctor; strive always to be that, an excellent doctor with a big heart

Han Sheng Thanks Dr Punna Wong for that. I also remember your lessons that the diagnosis can never be allowed to overshadow the patient - creating quotes like "That TB case" or "this Down Syndrome baby" when it should be - "that patient with TB" or "this baby with Down Syndrome". Whenever Dr Wong hears a student say "that case" he will inevitably ask - "What case? Pencil case?? Briefcase???"

Punna Wong I fervently hope that when you become a teacher/ doctor/ consultant you will remember these principles that I shared. I chant the same song over and over again. But these are songs worth singing, and I really hope that your generation will continue with these evergreens. Nc Huan When there is a new patient in the ward: Nurses: doctor, anyone clerking that new CASE? MOs: where is the new CASE? Most of my colleagues: what's the CASE about? You will hear things like this everyday: Oh that pharyngitis boy, oh that Downie girl etc Sad Medical knowledge and experience can accumulate as time passes, but for most medical graduates, the undergraduate period is perhaps the only time you learn about professionalism and patient respect Punna Wong If you are Mindful of this sad state, I have already taught you well. If you Mindfully always recall that there is a Human Being behind the Illness, I can retire in Peace knowing I have done some good in this world .

Ho Loon Shin I've learned from TJD that you do NOT have to be a perfectionist. TJD often tells students that when you are right 60% of the time you are a satisfactory doctor, 70% you are a good doctor, 80% you are a top diagnostician, and if you claim 100% then you must be a liar. The stats remind me that I (or you) can be wrong -- a good lesson for humility. Mario Yoong Primum non nocere Above all, do no harm! or First, do no harm. Some had attributed this to Hippocrates, but although the Hippocratic Oath did mention that we must do no harm when we practice the art of healing, this latin term was not actually used by Hippocrates. This term was used much later and its actual origin was never quite certain.

14. And the Face looks like...

This picture is at least 50 years old, a precious picture printed on a piece of glass kept by TJD. What is the name of this syndrome, its cause and its treatment? What is the prognosis?

Lee Chiang Sheng This picture depicts a lady with right LMN facial nerve palsy (loss of forehead wrinkle, eye closure, nasolabial fold). Multiple vesicles are over the external ear. Ramsay Hunt Syndrome/zoster oticus is the diagnosis. Reactivation of the varicella zoster in the geniculate ganglion is the underlying pathophysiology. The most important treatment is prednisolone to reduce the edema and inflammation of the facial nerve sheath to improve outcome. The use of acyclovir within 72 hours gives best outcome. Dosage is 800mg 5 times a day for a week. Relevant history are: eye dryness (due to loss of lacrimation), hyperacusis (due to loss of innervation to the stapedius; the 7th nerve contribution to the stapedius is responsible for dampening the vibrations of the stapes, thus controlling the amplitude of sound that is heard), loss of taste. Physical examination is mainly to differentiate UMN and LMN lesion as well to look out for other cranial nerve palsies (e.g. CPA tumor can affect CN 5,7,8). Places to examine are the ears (including performing a ear examination using an otoscope e.g. otitis media can cause CN7 palsy), parotid (for parotid tumor). The reason as to why someone with UMN CN7 palsy cannot grin when asked to do so but can smile when he/she hears something funny is because the UMN nerves receives innervation from the limbic system which is mediated by the nucleus accumbens in response to emotion. I remember this clearly when Prof Punna Wong was examining a stroke patient I presented. While doing cranial nerve examination it was clear that she can't grin and nasolabial fold was absent. She was clearly having difficulty raising her limbs when we needed to test for power but Prof kept on encouraging her and with every effort he commended and praised! That was when he directed our attention to her smiling face. Punna Wong Almost all of the cranial nerves receive bilateral innervation from the fibers of the pyramidal tract. This means that both the left and right members of a pair of cranial nerves are innervated by the motor strip areas of both the left and right hemispheres.

This is a safety mechanism. If there is a unilateral lesion on the pyramidal tract, both sides of body areas connected to cranial nerves will continue to receive motor messages from the cortex. The message for movement may not be quite as strong as it was previously but paralysis will not occur. The two exceptions to this pattern are the portion of CN XII that provides innervation for tongue protrusion and the part of CN VII that innervates the muscles of the lower face. These only receive contralateral innervation from the pyramidal tract. This means that they get information only from fibers on the opposite side of the brain. Therefore, a unilateral upper motor neuron lesion could cause a unilateral facial droop or problems with tongue protrusion on the opposite side of the body. For example, a lesion on the left pyramidal tract fibers may cause the right side of the lower face to droop and lead to difficulty in protruding the right side of the tongue. The other cranial nerves involved in speech and swallowing would continue to function almost normally as both members of each pair of nuclei still receives messages from the motor strip. On the other hand, unilateral lesions of the lower motor neurons may cause paralysis. This occurs because the lower motor neurons are the final common pathway for neural messages traveling to the muscles of the body. At the level of the lower motor neurons, there is no alternative route that will allow messages from the brain to reach the periphery. Muscles on the same side of the body as the lesion will be affected. Hence a 3rd CN palsy is a LMN lesion. Injuries to the nuclei of the cranial nerve nuclei located in the brain stem are called bulbar lesions. The paralysis that they produce is called bulbar palsy. It is usually bilateral. As cranial nerves are lower motor neurons, both bulbar and peripheral lesions are lesions of the final common pathway. When bilateral lesions of the upper motor neurons of the pyramidal tract occur, they produce a paralysis similar to that which occurs in bulbar palsy. For this reason, the condition is known as pseudo-bulbar palsy.

Hu Mung Chee Oral acyclovir 800mg five times a day for 7 to 10 days, famcyclovir 500mg tid for 7 days, or valacyclovir 1g po bid for 10 days and oral steroids (prednisolone 60mg/day) are mainstays of treatment. The prognosis of facial paralysis due to HZO is worse than that of Bells palsy. Approximately 10% and 66% of patients with full and partial facial paralysis, respectively, recover fully. The prognosis improves if the symptoms of HZO are preceded by the vesicular eruption.

To all my Students, "if you want to do Internal Medicine, you've got to live Internal Medicine, eat Internal Medicine, sleep Internal Medicine". "An uncommon presentation of a common disease is more common than the common presentation of an uncommon disease"

15. The Eyes are Windows

What is the diagnosis? What are the likely causes? How do you differentiate between "Medical" vs "Surgical" causes? What is the Nucleus involved that plays a role to help us differentiate between the two? Marcus Teng Lung En Patient shows a down and out position of the left side of the eye, and ptosis of the left eyelid. The diagnosis would be left oculomotor nerve palsy. It could be due to vascular disorder (ischemia caused by atherosclerosis, diabetes or hypertension), space occupying lesion such as a tumor, infections such as cavernous sinus thrombophlebitis. In order to differentiate between medical and surgical causes, one needs to assess whether the pupil of the affected eye has become dilated. Ischemia would spare the parasympathetic supply of oculomotor nerve while affecting the somatic innervation first, hence, the pupil may not be dilated, hence it's medical. However, a space occupying lesion would most likely affect the parasympathetic portion first, hence the pupil would be dilated early and it's a surgical cause. The reason would be because of the peripheral location of the parasympathetic supply on the oculomotor nerve.

Dr Patrick Cheah the parasympathetic fibres arise from the Edinger-Westphal subnucleus. The pupillomotor parasymparasympathic fibres running between the brain stem and cavernous sinus are located superficially in the superior median part of the 3rd nerve. Blood supply of this part comes from the pial mater vessels, whereas the main trunk of the nerve is supplied by the vasa nervorum. Thus: SURGICAL lesions eg, aneurysms, trauma, & uncal herniation involves the pupil by compression whereas MEDICAL lesions eg. hypertension, diabetes usually spare the pupil because the microangiopathy associated with these medical lesions involves the vasa nervorum, causing neural infarction of the trunk of the nerve, but sparing the superficial pupillary fibres Punna Wong How is it that a young chap playing Baseball who got whacked on his head by a hard ball flying at him at 100kmh will end up with a dilated pupil on that side of his head? What is the patho physiology? And will he have ptosis and a down and out eye??

Chee Yong Chuan A diagnosis of traumatic brain injury should be high up on the list. Epidural hematomas (EDHs) are blood clots that form between the inner table of the skull and the dura. Most EDHs are caused by direct impact injury that causes a forceful deformity of the skull. And in this patient, a solid, hard baseball could have caused a fracture of the skull injuring the middle meningeal artery that runs beneath the pterion which is the thinnest part of the skull, making it an extremely vulnerable to injury. The temporoparietal region is the most likely site for an EDH. The deterioration of a patient who has an EDH from arterial bleeding can be rapid and dramatic. Expanding high-volume epidural hematomas can produce a midline shift and herniation of the brain (transtentorial uncal herniation).

Dilated pupil on the site of the lesion can be observed because a transtentorial uncal herniation leads to compression of the 3rd cranial nerve which lies in close proximity causing an ipsilateral fixed and dilated pupil. Further uncal herniation compresses the pyramidal tract leading to contralateral motor paralysis (can be ipsilateral too-->Kernohan notch phenomena)

Pls see the position of the 3rd CN. 16. The EYES!

1. What do you see here? This patient had fever for 3 weeks and no one seems to know why! Careful examination revealed this

2. No its NOT dirt that the patient stepped on! Its painless and flat and the patient was not even aware of it. But she did complain of pain here:

3. Pls tie the whole picture together using the principle of Occam's razor!

Eng Kian Ng Picture 1: conjunctival haemorrhage of the left eye. Picture 2: Janeway lesions (no pain) Picture 3: Osler Nodes (pain) Possible DX: Subacute bacterial endocarditis

Hu Mung Chee Both found in bacterial endocarditis. Osler Nodes: Painful, palpable red lesions usually on fingers/toes. They are caused by immune complexes. Janeway Lesions: Non-painful, macular lesions, usually on palms/soles. They are caused by septic emboli Punna Wong What did Edward Janeway himself say!!!: His original description was in the context of separating "malignant" (acute) endocarditis from other infectious disorders. In an 1899 article published in Medical News,' he stated: In trying to determine whether a given case is more probably due to endocarditis or to another malignant process I have found that attention to the position of the hemorrhages is at times very helpful. Several times I have noted numerous small hemorrhages with slight nodular character in the palms of the hand and soles of the feet, when possibly the arms and legs had but a scanty crop in malignant endocarditis, whereas this has not been my experience with processes likely to be mistaken for it. And what did Osler himself say!!: In his Gulstonian. Lecture Series of 1885, Osler described the cutaneous manifestations of severe infectious endocarditis as "haemorrhage upon the skin, serous and mucous surfaces, in which "a minute necrotic or suppurative centre can sometimes be seen

Kianseng Ng Splinter haemorrhage is a nonspecific finding and can be associated with subacute bacterial endocarditis, scleroderma, trichinosis, Systemic lupus erythematosus, rheumatoid arthritis, antiphospholipid syndrome, and trauma. At first they are usually plum-colored, but then darken to brown or black in a couple of days. In infective endocarditis, they are probably caused by microembolisation to linear capillaries under the nails.

16. The Gums and Teeth What does this suggest to you? Let us think about the 3 most important possibilities

Lau Ron Hsien Gingival hypertrophy. In terms of Inflammation, the possible causes would be Periodontitis, Chronic gingivitis... Iatrogenically, three main groups of drugs: Anticonvulsants (Phenytoin, Na Valproate), Calcium Channel Blockers (Nifedipine, Verapamil) and Immunosuppresants (Cyclosporin). Lastly, due to Systemic diseases which include Scurvy, Wegener's Granulomatosis, Sarcoidosis, and in Acute Myeloblastic Leukemia.

This patient will have No Problem flossing his teeth. What could have caused this?

There is characteristic separation of the teeth of acromegaly. However if the patient has spaced out teeth without gigantism or acromegaly, then we must think of other possible causes of which a rare genetic condition is Ellis-von Creveld Syndrome, where patients would also present with polydactyly, cryptorchidism, and short stature with short limbs (think of it as the miniature equivalent of acromegaly in a patient with spaced out teeth).

Hutchinson's Teeth of Congenital Syphilis. Usually seen in children born of mothers with untreated Syphilis. The child would usually present with the Hutchinson's Triad, which includes Hutchinson's Teeth, Deafness, and Interstitial Keratitis.

17. The Tongue What would have caused this Unique Tongue?

Lam Jeng Hwan The tongue looks big and fills the oral cavity even when tongue is protruding out suggestive of macroglossia. Common causes would include: Acromegaly, Hypothyroidism, Down syndrome, amyloidosis The whitish tongue may be due to oral dryness due to macroglossia.

What is the abnormality here? Why is it that in patients with stroke the deviation of the tongue tends to recover after a few days? Lau Ron Hsien Left Hypoglossal Nerve Palsy (CN XII). Along with the tongue atrophy and if fasciculations are seen, we can conclude that this is a left lower motor neuron lesion. In a CN XII palsy, the tongue will deviate to the pathological side. This is because the Genioglossus muscle works in such a way where 2 muscle bellies lie next to each other and work together. As one gets paralysed, when the tongue is being protruded out, the normal side will extend, but nothing happens to the pathological side, which makes the tongue turn toward the pathological side. In stroke patients, it is usually a Upper Motor Neuron lesion, and since the CN XII has a small contribution from the ipsilateral side, the loss of function of one side of the tongue is transient and will typically recover in a few days after the stroke.

18. The neck

My students are familiar with looking for JVP, LNs and trachea deviation at the neck. But what if you see this instead!

What must you think of? Where else would you examine? How would you confirm the diagnosis?

Anver Sethwala Visible cutaneous nerve thickening, especially the greater auricular nerve, which also happens to be the only nerve better seen that palpated. Other nerves thickened and maybe palpable are the ulnar, supraorbital, supraclavicular, common peroneal, sural and median cutaneous nerves of the forearm. Also look for hypopigmented lesions around the body and test these for loss of sensation (anesthesia- temperature most sensitive) or autonomic nerve supply (loss of hair, sweat). There maybe autoamputation and nerve palsies eg: ulnar claw, wrist drop) secondary to the mononeuritis multiplex. Leonine fascies maybe present in the lepromatous type. SSS refers to the slit-skin-smear first discovered by Wade. This is a scraped incision as most of the mycobacteria are present in the dermis. No active lesion is mycobacteria free, and hence a mycobacterium index was discovered which measures the bacterial density of the live mycobacteria in an area. There is also the morphological index which measure the amount of dead mycobacteria and will still be positive despite treatment (as it takes time to clear the dead bacteria from the body by the macrophages). These two values are used to follow the progress of the disease as well the treatment effect. They need to be performed from the same area of skin (commonly ear lobe, and a fixed point on a lesion). Biopsy can also be done on the lesions to check for the mycobacteria. 19. Looking at the Neck may well tell you about the Heart! TJD took great pains to teach us the CVS Examination. He INSISTED that we be able to make a diagnosis based on History, Pulse, BP, looking at the Neck, looking meticulously at the praecordium, and feeling the Apex Beat and the praecordium. Many a times he playfully threatened to throw our stethoscopes out the 13th floor of UH when we needed it to make a diagnosis! He said "The best thing you can do to improve your CVS examination is to throw the stethoscope away!" I similarly insist that my students appreciate the various pulses, the JVP waveforms, the position and character of the Apex Beat. Palpable heart sounds, Thrills and size of liver all help us to make the Diagnosis BEFORE we place the stethoscope on the chest wall.

Listening is NEVER BLIND but guided by the knowledge accrued by the eyes that see and hands that feel.

For eg the diagnosis would be clear here before the stethoscope is placed on the chest Peripheral signs of AR for the astute clinician: 1. Quincke sign (pulsatile nailbed) 2. De Musset (head nodding) 3. Muller sign (pulsatile uvula) 4. Traube sign (a 'pistol shot' systolic sound heard over the femoral artery) 5. Duroziez sign (systolic and diastolic murmur heard over femoral artery on compression) 6. Rosenbach sign (liver pulsation) 7. Becker sign (accentuated retinal artery pulsation) 8. Gerhard sign ( pulsatile spleen) 9. Corrigans sign 10. Corrigans pulse Punna Wong Remember the Jugular Venous Pulse waves: a - wave : due to Atrial contraction x - descent : due to atrial relaXation c - wave : Bulging (Closure ) of Tricuspid valve into right atrium v - wave : Venacaval blood Volume increased when Tricuspid valve is closed y - descent: Tricuspid valve opening and inflow of blood into right ventricle

From this we can understand the Giant a Wave that occurs when the R Atrium contracts and the Tricuspid Valve happens to be closed in Complete Heart Block. We also understand why we see a Giant v Wave when the R ventricle contracts and there is Tricuspid Incompetence 20. LOOK CAREFULLY!

Can you pick up the Physical Sign shown here? A sufferer from angina, the great anatomist and surgeon John Hunter found that his attacks were often brought on by anger. He declared, 'My life is at the mercy of the scoundrel who chooses to put me in a passion.' This proved prophetic: at a meeting of the board of St. George's Hospital, London, he became involved in a heated argument, walked out of the meeting and dropped dead in the next room.

Will my life similarly be ended by Medical Students coming to class in shorts and slippers!?
What is the name of this sign? What does it imply? How sensitive and specific is it?

Eng Kian Ng Franks Sign, a diagonal groove at the level of the earlobe has been associated with Coronary heart disease, acting as an independent marker of coronary disease. The specificity of one study conducted in 1987 states the sensitivity of 75% and specificity of 57.5%. Lau Ron Hsien Seeing it in an elderly man may mean that he has IHD but at the same time, it may be normal with aging. Seeing it in a young person (20-40) would prompt us to be more vigilant in our search

21. We now look at the trunk

Can the students describe what are the abnormalities and the complications? What is the inheritance of this disorder?

This second picture is of a different patient. Do you know the Diagnostic criteria? Wong Wai Kit The pictures depict a patient with neurofibromatosis type 1. The first picture shows numerous neurofibroma on the back. It is autosomal dominant. The complications for NF1 are scoliosis, optic glioma, epilepsy, learning disability and hypertension due to association with pheochromocytoma. The diagnostic criteria for NF1 is 1. >6 cafe-au-lait spots (>15mm- after puberty, >5mm - before puberty) 2. 2 or more Lisch nodules 3. Axillary freckling 4. 2 or more neurofibroma 5. sphenoid dysplasia 6. positive family history (first-degree relative with NF1) 7. optic glioma The second picture shows a plexiform neurofibroma, affecting his face as well as his back. Kianseng Ng Complications...dont forget Malignant change, I just saw a gentleman with NF1 and he had a metastatic lesion in the lung. Neurofibromatosis type 1 (NF1) is an autosomal dominant disorder, the cardinal feature of which is the development of multiple peripheral nerve sheath tumors called neurofibromas.

Although neurofibromas are benign tumors, malignant peripheral nerve sheath tumors (MPNST) may occur. In addition, gliomas, particularly pilocytic astrocytomas of the optic nerve, and leukemias, are seen with increased frequency in the NF1 population. The gene responsible for NF1 has been cloned, and encodes a protein referred to as neurofibromin. Although the function of neurofibromin is not completely understood, it is known to include a GTPase activating protein (GAP) domain that regulates hydrolysis of Ras-GTP to Ras-GDP. This emerging understanding of the pathophysiology of NF1 has suggested new avenues of treatment involving the use of Ras inhibitors. Although only a minority of patients with NF1 develops malignancy as a complication of their disorder, cancer remains an important cause of morbidity and mortality in the disorder. 21. A Lesson in Medical Education well learned I was a Final year student and Prof Danaraj was teaching us at our weekly Wed Case Conference. The emphasis and discussion was mainly around the History, Phy Examination and Diagnosis, Differential Diagnosis and Investigations. Impatient as the young are likely to be, I asked Prof as to when we will learn about Therapeutics for we appear to regularly and repeatedly only discuss the "Basic stuff". Prof BOOMED "If your Diagnosis is wrong, what is there to talk about Therapeutics!!" Still unrepentant, I asked "But we are going to graduate soon, if we don't learn about Therapeutics now, when are we going to learn it?" TJD looked at me and sighed. "If we are to teach you everything you need to know BEFORE you graduate, you will then ONLY graduate at the age of 65!"

3 decades later, I still recall this conversation vividly. As a teacher now, I see the wisdom of dear TJD's words. Teaching medical students the Art of Diagnosis is the principle skill we must pass on to them. Ethics has to be learnt from example and apprenticeship. We cannot force ethics down their throats Peking Duck style. Much of Therapeutics is learnt as a Houseofficer, the undergraduate merely learning its broad principles; doses, adjustments, etc is best learnt during clerkship as a young doctor. No student will succeed in memorising all the doses and combinations from a textbook. Believe me I tried! And TJD was way ahead of the SDL (Self Directed Learning) so popularly preached today, he knew that it was IMPOSSIBLE to transfer the entire corpus of knowledge to us the undergraduate, much of it we must learn ourselves and upgrade as we go along, a lifelong learning process. We must 'reconfigure' our thoughts and approaches often to fit in with the state of the art in medicine much like my silly computer often interrupting my work as 'WINDOWS IS NOW UPDATING AND RECONFIGURING!'

Ho Loon Shin I remember this impt TJD's emphasis on getting the diagnosis right; for treatment, he says: just look it up in the textbook if you don't know. Kianseng Ng If our windows do not update and reconfigure, we will soon be practising cavemen medicine and windows will soon be saying, "cannot reconfigure, please sent pc to museum, expired!"

Daniel Eh Zhen Wei Thank you for sharing your experience with this great teacher. He must have made a great impact during his lifetime. A true hero and exemplary doctor whom we all aspire to be someday!

22. We must also look at the BACK of the Trunk! What can you see that will lead to your Diagnosis?

Lau Ron Hsien The tortuous and dilated veins over the back may be suggestive of an Inferior Vena Cava obstruction, which would cause the veins in the lower part of the body to be congested, and dilated, forming anastomoses with each other in an attempt to relieve the built up pressure...It is perhaps a chronic and progressive Inferior Vena Cava obstruction. Punna Wong The blood can also return via the veins at the back of the trunk. This picture was uploaded first to teach a basic lesson: LOOK AT THE BACK!

Perinephric abscesses, spina bifida, coarctation of the aorta are among the many that doctors had missed simply because he did not bother to look at the back of the patient! I now give you his Front!!

The Veins appear like snakes, Serpiginous. Arteries are straighter. We can also see it radiating out from the umbilicus. When you see snakes coming out of the centre radiating away, what does this picture remind you of? The cause is Obstruction of blood flow from the abdomen returning to the heart hence alternative routes usually minimally functional now becomes prominent. The cause of the obstruction is varied.

Mario Yoong The Medusa was an ugly creature, the daughter of Phorkys and Keto, the children of Gaia (Earth) and Okeanos (Ocean). She was one of the three sisters known as the Gorgons. She was once very beautiful and lived far in the north where the sun didn't visit. Being very curious, she wanted to see the sun, and asked the Goddess Athena for permission to visit the south. Athena refused to allow her to visit. The medusa got angry and dared to say that Athena hadn't given her permission because she was jealous of her beauty. Athena was angered and punished her by turning her hair into snakes and cursing her by making her so ugly that who ever looks at her eyes would turn into stone. In Medicine CAPUT MEDUSAE Sign (or the Head of Medusa) A term referring to the engorged veins that radiate from a recanalized umbilical veinfalciparum ligament in portal hypertensionwhich is most commonly seen in advanced cirrhosis, and is accompanied by ascites, hepatosplenomegaly, and patent hepatic veins Sashillia Raj An alternative version of the story: Medusa was once very beautiful.. And a devotee of goddess Athena... Therefore she vowed to remain celibate her whole life. However, Poseidon became aware of Medusa's beauty. One day, while she was praying in the temple, he raped her. Medusa prayed to Athena for help, but in Athena's eyes, Medusa seduced Poseidon... To add insult to injury, Athena turned Medusa into a hideous creature whose stare would turn anyone to stone. And so she became a recluse, living alone somewhere near the underworld Ong Lit Wei Dilated periumbilical veins radiating from the umbilicus reminds us of medusa's head full of snakes, hence the name caput medusae. To determine whether the cause of vein dilation is in the IVC or the portal vein, we would have to test for the direction of venous flow.

In caput medusae, the veins form a porto-systemic shunt, therefore the veins would flow away from the umbilicus. In IVC obstruction, veins dilate to return blood to heart. Therefore the venous flow would be upwards. We should test the direction of flow in the veins below the umbilicus. In caput medusae, the veins flow downwards and in IVC, the veins flow upwards. 23. A subtle sign on the trunk This sign when noted on the trunk will give you the diagnosis! This patient had fever for a week. What is the sign called? What is the sequence of appearance of rash from D1 of a fever!

Edwin Koh You've taught us to remember 'Really Sick People Must Take No Exercise'.

Rash appearing on D1 of a fever would be Rubella. D2 with Scarlet fever, SmallPox on D3, Measles on D4, Scrub Typhus on D5, none on D6 and Enteric fever on D7 Since this sign is on the trunk, the patient also having fever for a week, and the spots look red and macular, these are probably Rose spots present in Enteric fever

Punna Wong At approximately the end of the first week of illness, the fever plateaus at 39-40C. The patient develops rose spots, which are salmon-coloured, blanching, truncal, maculopapules usually 14mm wide and fewer than 5 in number; these generally resolve within 2-5 days. These are bacterial emboli to the dermis and occasionally develop in persons with shigellosis or nontyphoidal salmonellosis.

Arif Rahim The 3 'weeks' of Typhoid fever: First week: Usually asymptomatic, fever develops toward the end of the week accompanied by nonspecific symptoms (flu, malaise, some abdominal pain, adults get constipation) Second week: Hyperpyrexia as Dr Wong mentioned, less than 30% develop the Rose spots on the trunk. Third week (week of complications/nightmares): multisystem involvement, acute deterioration - GI: gastric perforation - CVS: shock - CNS: meningitis, encephalitis Have not come across one here in the peninsular, hence the gravity of the situation did not occur to me until I saw at least 4-5 patients during a short posting in Kuching, with 2 of them admitted to ICU (all around a week after developing fever).

24. A disastrous Rash on the Trunk

The biggest fear of any doctor in treating a patient with medication is a disastrous reaction like this. What is the diagnosis? What are the common culprit drugs and infections?

Kianseng Ng The second Image...Erythema Multiforme is a hypersensitivity reaction usually triggered by infections and drugs. It presents with a skin eruption characterized by a typical target (iris) lesion. The target (iris) lesion of EM has a sharp margin, regular, round shape and three concentrate color zones: Centre is dusky or dark red with a blister or crust Next ring is a paler pink and is raised due to edema Outermost ring is bright red. The single most common trigger for developing EM is herpes simplex virus (HSV). Mycoplasma pneumonia is the next most common trigger. Other Infections that trigger EM: Parapoxvirus (orf and milkers' nodules ) Herpes varicella zoster (chickenpox, shingles) Adenovirus Hepatitis viruses Human immunodeficiency virus (HIV) Cytomegalovirus Viral vaccines Many drugs have been reported to trigger EM including barbiturates, non-steroidal anti-inflammatory drugs, penicillins, sulphonamides, phenothiazines and anticonvulsants. The first image is a doctors nightmare and the two drugs that are responsible are Tegretol and Allopurinol, always warn the patient that at first sign of allergy they must be admitted for intensive treatment... Veman Lim Picture shows erythematous target-shaped skin lesions with over the chest and head with mucosal involvement suggesting Stevens-Johnson syndrome (SJS) or toxic epidermal necrosis (TEN), depending on the extent of skin involvement (SJS < 10% total body surface area, TEN > 30%).

The dermatologic manifestations of either toxic epidermal necrolysis or Stevens-Johnson syndrome constitute an emergency.

Toxic epidermal necrolysis, an acute disorder, is characterized by widespread erythematous macules and targetoid lesions; fullthickness epidermal necrosis, at least focally; and involvement of more than 30% of the cutaneous surface. Commonly, the mucous membranes are also involved. Nearly all cases of toxic epidermal necrolysis are induced by medications, and the mortality rate can approach 40%. Manifestations of Stevens-Johnson syndrome include purpuric macules and targetoid lesions; full-thickness epidermal necrosis, although with lesser detachment of the cutaneous surface; and mucous membrane involvement. As with toxic epidermal necrolysis, medications are important inciting agents, although Mycoplasma infections may induce some cases. The mortality rate is much lower than in toxic epidermal necrolysis, approaching 5% of cases. The prognosis in these diseases is largely a function of the degree of skin sloughing. As the percentage of skin sloughing increases, the mortality rate dramatically worsens. For unknown reasons, however, the disease process in some patients simply stops progressing, and rapid epithelialization ensues. For patients experiencing sloughing over a large area of their skin surface, the mortality rate is much higher. Together, Stevens-Johnson syndrome and toxic epidermal necrolysis may represent a spectrum of a single disease process. Stevens-Johnson syndrome may also have features of the dermatologic condition erythema multiforme. Only early transfer to and care in a burn unit has been demonstrated to decrease mortality. Coupled with early withdrawal of offending agents, this intervention is the best treatment that can be offered at this time.
Hu Mung Chee First figure shows widespread erythematous bullae, and mucositis of the conjunctiva and lips (mucosal involvement). Second figure shows target lesions on the hand. In addition to the general "skin failure," SJS/TEN is a systemic disease with involvement of the respiratory and gastrointestinal systems.

Life-threatening metabolite derangements, sepsis, respiratory failure, and gastrointestinal hemorrhage may occur and are compounded by underlying comorbidities. All of the patient require admission to a burn intensive care unit. 25. A sign on the Upper Trunk What is this sign and what are its implications?

Tze Liang Loh Spider naevi suggestive of chronic liver disease


Punna Wong Spider naevi may also occur in healthy individuals. It seems likely that spiders and palmar erythema in patients with liver disease are a manifestation of disturbed circulating sex hormone activity, such as an elevated estradiol/free testosterone ratio or they may be a toxic result of alcohol when this is the cause of impaired liver function. Spider naevi are more common in patients with alcoholic liver disease than with other causes. 15-20% of healthy individuals will have one or more spider naevi Multiple spider naevi are indicative of underlying disease. Spider naevi may also develop in pregnancy, or when taking the oral contraceptive pill They are small lesions with a number of tiny blood vessels radiating from them to resemble the legs of a spider.

Almost all spider naevi occur on the upper part of the body and only 1% below the umbilicus. 26. A sign and a basic Ix often tells a lot! This young Chinese girl has Multiple recurrent Ulcers of her lips. They are occasionally Painful. No ulcers is noted in her genitalia, or rashes on her palms or feet. She has been married for 5 years and has not conceived despite unprotected sex. A simple basic ESR at her small town revealed 105mm/hr. What could be the diagnosis? What other signs will you look for? What further Ixs will you order?

Vinayak Smith SLE with antiphospholipid syndrome... Test for anticardiolipin and lupus anticoagulant... Thats what is probably causing her infertility via recurrent miscarriages Kianseng Ng Mucous membrane involvement occurs in 12 to 45 percent of patients with SLE. 1. Characteristic discoid lesions with erythema, atrophy, and depigmentation of the lips. 2. Gingivitis, particularly in individuals with xerostomia.

3. Irregularly shaped raised white plaques, silvery white scarred lesions, areas of erythema, and punched out erosions or ulcers with surrounding erythema may appear on the soft or hard palate or buccal mucosa. Palatal ulcers being most specific for SLE Oral lesions may be the first sign of lupus and they are usually painless. Wong Ling Ying A common differential is Herpes Labialis Vishnu Prashad Infertility in SLE may be due to the disease itself or as a result of treatment (especially with cyclophosphamide). Autoimmune mechanisms may contribute to premature ovarian failure & menstrual disturbance (which may also be secondary to medications used to treat SLE). Also, it is suggested that APS (which is more commonly associated with recurrent miscarriages, stillbirths and premature births as well as thrombotic events) may also have an earlier impact on fertilization, implantation and reproductive function as a whole. Anti-phospholipid syndrome (APS) occurs secondary to SLE in ~ 20-30% of patients and more commonly, as a primary disease. It is characterized by 'CLOT': Coagulation defects, Livedo reticularis, Obstetrics (recurrent miscarriages) and Thrombocytopenia. The antibodies which indicate presence of APS include the lupus anticoagulant and anti-cardiolipin antibodies.

27. A terrible Complication

This patient had a spontaneous L Pnemothorax. A Chest tube was inserted. A few days later, this was how the patient looked like! What had happened!?

This is his ECG What are the causes of this common ECG abnormality? What simple word helps you to remember the causes?

Nada Syazana s/c emphysema as a result of chest tube insertion causing high pressured air from pneumothorax to escape into the subcutaneous layers at the puncture site causing swelling at the chest, neck and sometimes abdomen. Can be left alone unless causing significant dyspnoea/airway obstruction then incisions or slits into the subcutaneous layer is made to let air escape. ECG demonstrate wide QRS complex with rsR pattern in V1 to V3, with T wave downgoing, and slurred S wave over V5 to V6. This is typical of right bundle branch block. Causes: IMRAN I = ischemia/infarct M = myocarditis R = RV strain e.g. RVH, cor pulmonale, PE A = ASD (ostium secundum) N = normal variant Kianseng Ng Normal Variant... (1) There are some medical authorities who believe, that RBBB (in the absence of diseases denoted in IMRAN) is "not really that benign and not that normal". (2) If a patient has a RBBB, always ask if he had a previous ECG. And if there was one and it was normal search for a cause...in the process of so doing you may uncover the not so common causes of RBBB, like recreational drugs esp Marijuana. Many ECG Abnormalities have been Reported with Marijuana use: Sinus Tachycardia, Supraventricular Tachycardia, Sinus Bradycardia, PVCs, Atrial Fibrillation, Atrial Flutter, AV Blocks, RBBB. Punna Wong RBBB occurs in medical conditions that affect the right side of the heart or the lungs, so a finding of RBBB ought to trigger a screening exam for pulmonary embolus, chronic lung disease, cardiomyopathy and septal defects. Ischaemia and Myocarditis may also cause it. However, RBBB also commonly occurs in 'normal, healthy' individuals, and screening often turns up no medical problems. In these patients, the RBBB has no apparent cause as far as we can see, its medical significance unknown, and is often written as a normal variant, and ignored.

More importantly, RBBB occurs in Brugada syndrome. Brugada syndrome is characterized by right bundle branch block and coved-type ST segment elevation (Shark Fin!) in the right precordial leads on 12-lead ECG and sudden death following ventricular fibrillation Hu Mung Chee For emergency patients, new onset RBBB suggests acute right heart strain, as may occur with pulmonary embolism. Paul Ling Kah Hing I get referrals of young people with RBBB and normally I do an echo to exclude right heart pathology and if the echo is normal I call them normal variant. But Dr Ng is right, they might not be as benign. Patients with pRBBB or RBBB must undergo careful examination for cardiac pathology. Kianseng Ng Brugada syndrome is a disorder characterized by sudden death associated with one of several electrocardiographic (ECG) patterns characterized by incomplete right bundle-branch block and ST elevations in the anterior precordial leads. Subtle structural abnormalities in the right ventricular outflow tract have been reported. Sudden cardiac death has a much higher prevalence in Southeast Asian countries, and has gone under different names such as bangungut (to rise and moan in sleep) in the Philippines, pokkuri (sudden death) in Japan, & lai tai (death during sleep) in Thailand, non-laitai in Laos. Brugada Signs in the ECG are dynamic & not static. The Type I ECG can change from and to Type 2, Type 3, or Normal ECG spontaneously or under the influence of Hyperthermia or drugs. If clinical circumstances arouse strong suspicion that a patient may have Brugada Syndrome & the ECG is normal, repeat the ECG but move V1-3 upwards by 1 or 2 intercostal spaces. This simple procedure will unmask the Brugada signs if present. The rationale behind the upward relocation of the precordial leads is because the Brugada Signs are a reflection of conduction delay localized in the RV Anterior Wall & the RV outflow tract. The higher ECG leads would then cover the sites where abnormal repolarisation prevails.

Hu Mung Chee This is the ECG of the patient who presented to ED with syncope. Here we can see the incomplete right bundle-branch block and ST elevations in V1 & V2, suggesting Brugada syndrome.

Stress, Fever, Sepsis may unmask an Underlying Pathology. This patient has a Normal ECG (above) when he is well. When he had Acute Appendicitis, this was his ECG. ECG changes can be transient with Brugada Syndrome and be unmasked by multiple factors.

This ECG demonstrates the classical Brugada Sign of Type 1 Brugada syndrome. Coved ST elevation more than 2mm in V1 and V2 followed by an inverted T wave. ECGs courtesy of Dr Ong Jiann Ruey

Punna Wong IMRAN Khan was a legendary Pakistani cricketer whose name is immortalised in the causes of RBBB! Credit goes to Prof Esha Das Gupta my colleague in the Dept of Medicine 2 decades ago who found this man irresistibly handsome and came out with this acronym.

Imran Khan And Pakistan has a new fast bowler, Aizaz CHIMA! Now how does this new star help you to remember another ECG abnormality!?

Ng Hui Mien Left bundle branch block:

CHIMA

C - Cardiomyopathy H - Hypertension I - Ischemic heart disease M - Myocarditis A - ASD ostium primum, Aortic Stenosis

AMI in background of LBBB 28. No Eye SEE

Yesterday, Dr Ho Loon Shin joined me for one of my Tutorials. He must have been shocked when he listened to the "Focused" history and the "Focused" Physical Examination being presented! Gone are the days when the History was total and COMPLETE! Gone are the Days when the patient was examined from head to toe, from Neuro to Genital! Osler, Danaraj and many other great Clinicians are certainly turning in their graves/urns. Nowadays students are examined for their ability to take a "Focused History" or a "Focused Examination".

All these pictures are possible presenting signs of a common disease. Please LOOK for them and at them! What do they show? What condition can give rise to all of them as either presenting features or complications? Wong Ling Ying Diabetes Mellitus Pic 1: Fungal infection which is common in webs of fingers/feet, skin folds Pic 2: Macular star (can present in neuroretinitis, hypertensive retinopathy or DM retinopathy) and cottonwool spots on the medial side Pic 3: Candida infection of the penis

29. All about Nails

What is this sign and what causes it?


Bee Shuang Lee yellow nail syndrome can be found in DM, amyloidosis, median/ ulnar nerve injury, thermal injury, jaundice and in lymphedema, bronchiectasis

At Bedside today, we saw a patient with this sign. What is it indicative of and who is this sign named after? Tommy Chong Thoong Onn Half and half nail: Lindsay's nail. It is the diagnostic sign for ESRF. Dialysis will not influence the abnormality, however with successful renal transplantation the abnormality may disappear 2 - 3 weeks after the transplantation.

To see this sign requires OBSESSIVE examination! But it tells you a lot!

Nada Syazana Periungal telangiectasia. Causes: RA, SLE, dermatomyositis, scleroderma.

Tell me some causes of this sign


Euvin Teoh Raynaud's phenomena precipitated by cold environment. Can be found in: Normal healthy women (familial, <30 years) Connective tissue diseases eg. SLE, scleroderma (CREST) thoracic outlet obstruction, also Buergers, Takayasu's, trauma, prolonged exposure to cold.

This patient has lower backache and has seen many chiropractors and Orthopods. What is the diagnosis?

WanAmalyna WanFadzil Pitting of the nails- psoriatic nail changes. The patient probably has psoriatic arthritis affecting the spine. 30. Painful Shoulders

Your 60 years old aunt complains of pain around her shoulders, back and hips. It is also stiff and can sometimes swell and is worse first thing in the morning but eases as the day goes on. Occasionally her hands, feet and joints can swell too. She feels tired, depressed and experience night sweats, loss of appetite and occasionally fever. What simple Investigation MUST you do?

What is your clinical diagnosis? What do you expect the ESR to be? Carol Goh Polymyalgia rheumatica which has association with giant cell arteritis prevalent in women aged 50 years and above, who presents with morning stiffness which lasts 30 minutes or more with limitation in terms of functionality eg trouble dressing etc. Commonest sites of involvement are usually shoulder followed by hip and neck involvements. Interestingly, muscle strength is usually normal in this condition. However, patients may report weakness and careful examination to confirm muscle weakness is necessary in order to prevent misdiagnosis.

5 causes of ESR >100mmhr: multiple myeloma, polymyalgia rheumatica, TB, lymphoma, septicemia, autoimmune diseases, advanced malignancies. IM SMART I Infections like TB, Septicaemia M Multiple Myeloma S SLE, Auto Immune diseases M Malignancies when advanced with secondaries ART Arteritis esp Temporal Arteritis, Polymyalgia Rheumatica

1 = Normal ESR 2 = Normal ESR with reddish plasma in hemolysis (disease or artifact) 3 = Blurring of the plasma-erythrocyte border in reticulocytosis 4 = White turbidity and blurring in severe leukocytosis of leukemia 5 = Accelerated ESR and lipemic plasma after a fatty meal 6 = Accelerated ESR and icteric plasma 7 = "Zero" ESR in polycythemia 8 = Severely accelerated ESR in multiple myeloma Punna Wong The ESR is an inexpensive, simple test of chronic inflammatory activity and its follow-up. The ESR rises with age, but this increase may simply reflect a higher disease prevalence in the elderly. The use of the ESR as a screening test in asymptomatic persons is limited by its low sensitivity and specificity.

An elevated ESR is a key diagnostic criterion for polymyalgia rheumatica and temporal arteritis, but normal values do not preclude these conditions. An extremely elevated ESR (>100 mm/hr) will usually have a important causemost commonly infection, malignancy or temporal arteritis. A mild to moderately elevated ESR without obvious etiology should prompt repeat testing after several months rather than an expensive search for occult disease. In oncology, a high ESR has been found to correlate with overall poor prognosis for various types of cancer, including Hodgkin's lymphoma, gastric carcinoma, renal cell carcinoma, chronic lymphocytic leukemia, breast cancer, colorectal cancer and prostate cancer. In patients with solid tumors, a sedimentation rate greater than 100 mm per hour usually indicates metastatic disease. In Hodgkin's disease an elevated ESR may still be an excellent predictor of early relapse, especially if the value remains elevated after chemotherapy or fails to drop to a normal level within six months after therapy.

PRIMUM NON NOCERE!!

31. Tell me about patients presenting with Ischaemic stroke and BP of 200/110. What should the Emergency dept do? Chee Yong Chuan: whether or not to lower the BP is dependent on whether the patient is eligible for fibrinolytic therapy. Patients who are candidates for fibrinolytic therapy should have their blood pressure controlled to lower the risk of intracerebral hemorrhage following administration of rtPA. If patient is eligible for fibrinolytic therapy, blood pressure must be less or equal to 185mmHg systolic and less or equal to 110mmHg diastolic to minimize the risk of bleeding complications. If the patient is not eligible for fibrinolytics/or fibrinolytic therapy not available), consider lowering blood pressure in the setting of acute ischemic stroke if systolic is above 220mmHg and diastolic >120mmHg. This is based on the AHA 2010 recommendation, it is important to remember that the elevated BP serves to perfuse end organs (I.e the brain).

(In the setting of an Ischaemic stroke, the dead focus is surrounded by an Ischaemic Penumbra that the Brain is trying to perfuse by elevating the perfusion pressure. This auto regulation is a desperate preservation reflex to preserve the neurons. Lowering the BP may extend the area of Infarct).

But if there is presence of end organ injury like AMI, CHF, aortic dissection, consider lowering the blood pressure. A reasonable target is to lower blood pressure by 15% to 25% within the first day. However most patients in Malaysia are not eligible for fibrinolytic therapy: 1) Very costly 2) Patients will need to present within 3 hours of onset of symptoms. Very difficult in remote areas as most of them will need to be transferred to the nearest tertiary centre/stroke center 3) TPA will need to be given with ICU backup and presence of experienced neurologist Lee Chiang Sheng: Lowering of BP is only indicated if it's more than 220/120 mainly to allow the body's cerebral autoregulation to preserve the perfusion of the ischemic region which is already oxygen depleted...So a high BP in the initial phase is accepted. Hui Mien: There was in the ward a patient whose stroke got worse because of the doctors lowered the patient's BP. The patient eventually passed away. The specialist talked about it in the ward and I will remember it forever.

Yu Yen Inclusion guidelines for fibrinolytic administration within 3hrs of ischaemic stroke: 1. Ischemic stroke is diagnosed and caused measurable neurologic deficit 2. Neurologic signs don't clear spontaneously 3. Neurologic signs not minor / isolated 4. No symptoms that suggest subarachnoid haemorrhage 5. Onset of symptoms <3hrs before starting treatment 6. No head trauma /prior stroke / MI in last 3 months 7. No gastrointestinal / genitourinary bleed in last 21 days 8. No arterial puncture in non-compressible site in last 7 days 9. No major surgery in last 14 days 10. No history of intracranial bleed /acute trauma or bleeding 11. Systolic BP<185mmHg, diastolic BP<110mmHg 12. Not taking oral anticoagulant / if taking, INR<1.7

13. Normal aPPT if take heparin within 48hrs 14. Platelet > 100,000/L 15. Blood glucose > 2.7mmol 16. No seizure with residual postictal impairments 17. No evidence of multilobar infarction (hypodensity over 1/3 hemisphere) in CT Also show benefit if administer in 3-4.5hrs after stroke, but if with any 1 of the following, they should be excluded: 1. > 80 year-old 2. Take oral anticoagulants (regardless of the INR) 3. Baseline NIHSS score >25 4. History of stroke and diabetes 32. Taking a step back and Look at the patient

What is this sign? What does it indicate? Xinyi Yeow Gower's sign - Duchenne muscular dystrophy.

Gower's sign is when the child needs to turn prone to rise (early feature) and then uses his hands to climb up his knees to stand (late feature), due to poor pelvic girdle fixation and proximal muscle weakness. Turning prone to rise is normal in children up to 3 years. Bee Shuang Lee Duchenne muscular dystrophy, a X-link-recessive inherited disorder. They will have progressive loss of muscle strength and eventually unable to walk. Contractures of the muscle will lead to the development of progressive scoliosis and chest deformity, and hence impairing pulmonary functions. The characteristic ECG changes are tall R in V1 and Q in precordial leads. Intellectual impairment is common in this group of patients. 33. The Entire Body now! Look at this series of Photos. Pls comment on what you see.

What can you find on auscultation? What is the prognosis?

Ng Hui Mien 1. long thin feet, mobile pes planus due to hyperlaxity 2. high arched palate 3. tall, thin with long limbs 4. increased arm span to height ratio Auscultation may reveal murmurs if there is mitral regurgitation, aortic regurgitation. These patients are prone to aortic dissection or mitral valve prolapse.

Yeoh Seng Hong 1. Definition of Marfans syndrome: autosomal dominant inherited disorder of connective tissue characterised by loss of elastic tissue, affects numerous body systems. 2. Positive Walker sign-Distal phalanges of the first and fifth digits of the hand overlap when wrapped around the other wrist.

3. Positive Steinberg sign-When the patient bends the thumb toward the palm of the hand and covers it with the fingers, the tip of the thumb protrudes beyond the palm of the clenched hand. 4. Clinical presentations in the photos: tall stature wide arm span - Arm span-to-height ratio >1.05 high level of pubic bone -Pubic bone-to-height ratio >0.5 high arched palate arachnodactyly 5. Other expected or relevant findings that in other marfan's patients: pectus excavatum (funnel chest) pectus carinatum (pigeon breast) flat feet (pes planus) - ligament laxity. dislocated/subluxed eye lens myopia and/or astigmatism Anisometropia, amblyopia, or strabismus may occur. retinal abnormalities include myopic posterior staphyloma, lattice degeneration, white-without-pressure, retinal pigmentary changes, atrophic holes or retinal tears, choroidal and scleral thinning.. joint hypermobility 6. aortic valve regurgitation- Present in about one third of adults. mitral valve: Approximately one third of people have a nonejection systolic click and about 10% have a regurgitation murmur. 7. Prognosis: Marfans syndrome patients need long term follow up and surgery is indicated when the aortic root diameter in the sinus Valsalva > 4.5cm-5.0cm, and emergency surgery for aortic dissection. Pharmacological intervention such as introducing beta-blockers is favourable for long term survival. Punna Wong What is the difference between being "Marfanoid" and Marfan's syndrome?

Matthias Lu Zhuang Hao Being marfanoid basically means having physical features of Marfan's but not actually having the disease and the gene mutation. Some of the features are high arch palate, a positive Walker and Steinburg sign, and an increased arm-span to height ratio. However they would probably not have the other complications which would be due to the genetic defect as seen in Marfan's, leading to further complications such as retinal detachment, increased risk of aortic dissection, an AR, etc.. Punna Wong The constellation of long limbs, dislocated lenses and the aortic root dilation are generally sufficient to make the diagnosis of Marfan syndrome with reasonable confidence. Genetic testing is of importance in the 2010 criteria but here in Msia, the diagnosis is a clinical one. Distinguishing among "marfanoid" people for Marfan's can be facilitated by genetic testing if one is suspicious of more than just "body and skeletal" changes, or when there are signs and symptoms other than just body habitus. Marfanoid people have certain characteristics of Marfan and Ehlers-Danlos syndromes like long limbs, arachnodactyly, hyperlaxity and skin elasticity but without the terrible aortic, valvular and lens complications.

34. Massive Splenomegaly

An enlarged spleen is a common finding But Massive Splenomegaly indicates a few Important Possibilities. What are the Important causes in Malaysia? Matthias Lu Zhuang Hao A spleen that has enlarged to the extent of the midline and beyond is considered to be massively enlarged. A few important causes are 1. Malaria 2. Thalassemia major 3. CML 4. Myelofibrosis 5. Kala-azar The important ones in Malaysia are the first 4 listed.

Rare causes in Malaysia: Amyloidosis, Gaucher's, Sarcoidosis Splenic hydatid cyst Schistosomiasis Visceral Leishmaniasis
Kong Wah Ng Perhaps an easier way of remembering the causes of splenomegaly is to subdivide into (a) huge - enlarged across midline, past the umbilicus (b) moderately large - midway bet huge and (c) which is just palpable. Causes of huge spleen are CML, myelofibrosis, malaria and Kala-azar.

Causes of moderately large spleen are the above 4 plus storage diseases, haemolytic anaemias, portal hypertension and leukaemias. Causes of a just palpable spleen are the above 8 plus all other causes of splenomegaly

35. An Irregularly Irregular Pulse There are many causes of Bilateral Leg Swelling. In an elderly person with leg swelling and breathlessness, the finding of an irregularly irregular pulse tells us much about the possible pathology. What are the causes of an Irregularly Irregular pulse?

Lincoln Lim Jian Rong Common causes of pitting edema with breathlessness are CCF secondary to DM or long standing poorly controlled HPT or Ischaemic heart disease causing decompensated cardiomyopathy.

Daniel Eh Zhen Wei Other causes of pitting edema can be due to venous stasis due to prolonged immobility or standing, right heart failure, constrictive pericarditis, liver cirrhosis, nephrotic syndrome, nephritic syndrome. More rarely, GI causes include starvation, protein losing enteropathy and malabsorption syndrome. Not forgetting drugs such as steroids and CCBs. For non-pitting odema, it may be to due to myxedema, lymphedema, and also lipoedema. Lee Chiang Sheng There are a few causes for irregularly irregular pulse: 1) Atrial Fibrillation (commonest) 2) Multifocal Atrial Tachycardia (Usually in the setting of COPD) 3) Atrial flutter with varying block 4) Multiple ectopics 5) Sinus arrhythmia

Tommy Chong Thoong Onn Causes of AF HITAM (black colour) H Hypertension I Ischaemic heart disease T Thyrotoxicosis A Alcoholism M Mitral valve disease I learned this during last Thursdays bedside with Dr. Wong. Thanks to Dr Wong :) Sexually Transmitted Diseases Medical students exposure to STDs is minimal in a ward setting other than for HIV patients. HIV is seen at every ward nowadays. The students must overcome their initial fear, we all fear at first so there is nothing to be ashamed of. With time, we understand that it is just another illness that needs our attention. Think of the 19th century doctors handling patients with TB; they called it 'Consumption' then for they have NO CLUE what was killing these patients slowly.

Our Profession puts the patients above self; yes it is the principle behind why doctors served in Leprosy colonies and sanitariums, why doctors work in Biohazard Level3 laboratories. Silly this dictum may sound to a new generation but I believe all potential students must realise this before they enter medical school. If not, you are a poor candidate.

36. South of the Border Prof Danaraj once recalled to us that when he was a young doctor he had to run a STD clinic in the evenings where he saw scores of patients within a very short time. It was literally a patient walking in, dropping his/her pants, the doctor looking, scribbling a prescription and off the patient went. He asked us "How do you think I performed as a doctor then?" We all kept quiet. He said "It was TERRIBLE!" It is indeed humbling to us when the great Professor himself told us that the environment and circumstances could make even the best of us underperform. The University Hospital was built to provide medical students with the best possible environment and an IDEAL to look forward to. It was to challenge us to raise our standards beyond the mundane and for us subsequently working in the real world to try and elevate standards. Here are some common problems: A patient complains that he got "WHITE PORRIDGE "

Being a good doctor, you insisted on examining his wife and sexual partners. This is what you found What is the diagnosis? How do you confirm it? How should you manage the patient? Tatsuki Abe Gonorrhea. - Confirmed by demonstrating gram-negative intracellular diplococci. Rule out concomitant infection with other STDs. We would need to council the patient and his partner to refrain from sex until both of them are clear of infection, and to advise faithfulness to each other... - if not, use protective measures! "If you can't be good, be safe". Screening for HIV after 3 months is needed.

For the uninitiated, Intracellular G-ve diplococci The small dots in the cells!!!

Anver Sethwala Cultures with Thayer-Martin medium are to be taken from the nasopharynx (oral sex), urethra, endocervix (vaginal sex) and anus

Punna Wong C&S is not frequently done and treatment is based on clinical diagnosis most times; as it is NOT possible to tell if there is Gonococcus + Chlamydia infections, co treatment for both is often given. A 10 days course of Doxycycline is given on top of whichever drug that is chosen for the treatment of the Gonococcus. A single dose of IM Ceftriaxone followed by the Doxy is a common choice.

Women are SPECIAL! The majority of urethral infections caused by N. gonorrhoeae among men produce symptoms that cause them to seek treatment soon enough to prevent serious sequelae but treatment might not be soon enough to prevent transmission to others. Among women, gonococcal infections might not produce symptoms until complications (eg. PID) have occurred. Women can also be Asymptomatic carriers spreading to Boyfriends, Clients and Neonates: A modern Gonococcus Mary! Hu Mung Chee Cotreatment for Chlamydia trachomatis (azithromycin 1 g orally once or doxycycline 100 mg orally twice daily for 7 days) is recommended for individuals infected with Neisseria gonorrhoeae and may reduce the development of antibiotic resistance to N gonorrhoeae. Because of antibiotic resistance, quinolones are not recommended as treatment for N gonorrhoeae. Because of concerns about emerging resistance to cephalosporins and their efficacy against potentially unrecognized gonococcal pharyngeal infections, ceftriaxone 250 mg intramuscularly once (instead of the previously recommended 125 mg) is now the recommended treatment with uncomplicated infections for adults and for children who weigh greater than or equal to 45 kg (125 mg for children <45 kg). Cefixime 400 mg orally is an alternative for adults but not for children, does not have as high or as sustained a bacteriocidal level as ceftriaxone, and is less effective against gonococcal pharyngeal infections, which are commonly asymptomatic. Certain other cephalosporins are potential alternatives (eg, ceftizoxime, cefoxitin, cefotaxime) but have uncertain or less efficacy against gonococcal pharyngeal infections. Spectinomycin is effective against urogenital and anorectal gonococcal infections, has poor efficacy for pharyngeal infections. Azithromycin 2 g orally is active against N gonorrhoeae but is not recommended because of concerns about treatment failures and development of antibiotic resistance.

37. South of the Border, Down by the Sea

This is painful and recurrent. The vesicles break and shallow ulcers results. In the female, there is risk to the newborn as well. How do we manage this common condition? Wong Ling Ying Genital Herpes. Caused by HSV-2 (HSV-1 causes cold sores, affects mouth and lips) HSV-2 is spread via secretions and sexual contact. Antivirals such as acyclovir, famciclovir and valacyclovir can be given to relieve pain and help prevent outbreak or shorten the duration. The patient remains as carrier for life and is never cured from the infection. Pregnant woman with active genital herpes infection are at risk of premature birth and the fetus may develop meningitis/encephalitis, chronic skin infection, developmental delay or death. Therefore women with active disease are advised for C-section instead of vaginal delivery.

Punna Wong The herpes virus can be active on the surface of the skin without showing any signs or causing any symptoms. So even if it LOOKS normal it may NOT be normal!

Condoms provide only partially protect against herpes virus infections. Condoms reduce the risk of transmitting genital herpes, however, they do not entirely remove the risk. Condoms may not cover all the areas where the herpes virus may be present, so they aren't 100% effective in preventing the spread. The number of sexual partners is not a factor in contracting the disease as, contrary to other STDs, genital herpes persists indefinitely and can be transmitted for many years, perhaps for life. 20% of those infected with HSV-2 experience no signs or symptoms and do not realise they have the virus. Note that HSV1 that causes "Cold sores" can be transmitted during oral sex and can cause genital herpes. The ONLY SAFE SEX IS NO SEX! Sad but the truth 38. Awesome painless craters

"By the way doc, can you look at an ulcer in my penis".

His wife came 3 months later, and this is what you found! What is the problem afflicting the above patients? Lau Ron Hsien The first two pictures depict a solitary ulcer with an indurated border. And considering the fact that the patient did not complain of pain, it is highly suggestive of a chancre caused by Syphilis. However, if the ulcer is painful and weeping, producing purulent discharge, then one must think of a chancroid caused by the infection of Haemophilus ducreyi. RPR and VDRL are non specific, as farmers working in the fields will also test positive, as they have been exposed to different strains of the Treponema spirochaetes. TPHA on the other hand, is specific for Treponema pallidum, the causative agent of Syphilis. Contact tracing would be required, as partners would have to be screened and treated accordingly. Do note that in Malaysia, it is also a notifiable disease, so paperwork has to be done accordingly. For primary (Incubation: 10-90 days, avg 21 days) and secondary Syphilis, treatment is with Benzyl Penicillin suspension IM. If the disease is not treated and allowed to progress, the chancre will heal regardless of treatment and following that, a maculopapular rash will present itself after one to 2 months. The unique thing about these rashes is that they are one of the few rashes to involve the palms and the soles. And also at mucous membranes, there will be the presence of condylomata lata, which is highly infectious, seen on the patient's wife. However, the moment eye signs are observed, namely the ArgyllRobertson Pupil (a prostitute accommodates but does not react) then the disease is deemed to have progressed further into tertiary Syphilis, Neurosyhilis. Neurosyphilis is characterised with 2 big presentations, Tabes Dorsalis and General Paresis of the Insane. This stage of Syphilis only occurs if the disease is allowed to progress for years. We may also see Syphilitic Aortic Aneurysm of the ascending aorta.

Lincoln Lim Jian Rong Patients with open syphilitic sores have a 2-5x increased risk of contracting HIV through unprotected copulation.

Courtesy Dr Hu Mung Chee Prior to the introduction of penicillin in the 1940s, syphilis was treated primarily with heavy metal injections eg Bismuth, Mercury. This radiograph shows bismuth injection granulomas in the gluteal muscles. One Night with Venus, a Lifetime with Mercury!

Dr Patrick Cheah: 39. Blood Spillage & HIV patients

HIV is fragile and does not survive long outside the human body Studies performed at CDC have also shown that drying HIV causes a rapid within several hours 1-2 log (90%-99%) reduction in HIV concentration. HIV can be transmitted through contact with infected Human blood, Semen, Vaginal secretions, Internal body fluids, Saliva in dental procedures, Any body fluid with visible blood The following body fluids are not considered infectious for HIV: Tears, Sweat, Vomitus, Feces, Urine HIV are not spread through food or water, sharing eating utensils or water glasses, or an infected person sneezing or coughing on you HIV are not spread by mosquito or other insect bites. HIV are most commonly transmitted through Sexual contact Sharing needles illegal drug use From mothers to their babies at/before birth The likelihood of being infected with HIV from a single stick with a HIV contaminated needle is < 0.5 %. In contrast, the likelihood of being infected with HepBV from a single stick with a HepBV contaminated needle is 6-30%.

There are three potential routes of transmission of a HIV from job related tasks: 1. Accidental puncture from contaminated sharp object 2. Contact between broken or damaged skin and infected fluids 3. Contact between mucous membranes and infected body fluids You may have been exposed to a Blood Borne Pathogen on the job if ---Blood or other potentially infectious material splashes in your eyes, nose or mouth. ---Blood or other potentially infectious material has contact with an opening in your skin (e.g. cut, rash) ---A sharp object (e.g. needle) contaminated with blood or other potentially infectious material breaks your skin.
40. Cauliflowers down South

The diagnosis is obvious. Is there a risk of Malignancy? Can we prevent this?


Philip Pang It's a genital warts caused by hpv HPV is a group of more than 100 related viruses. HPV-16, HPV 18, HPV 31, HPV 33 and HPV 45 are considered high-risk types of HPV because they are strongly linked to cancer. High-risk onchogenic HPVs cause virtually all cervical cancers and most anal cancers and some vaginal, vulvar, penile, and oropharyngeal cancers. Other subtypes of HPV can cause warts in the genital and anal areas: condyloma acuminatum. HPV types 6 and 11 cause 90 percent of all genital warts. They are called low-risk types of HPV because they tend to cause warts but not cancer. HPV infection can cause anal and genital warts, but most people infected with HPV do not have genital warts or any other signs of infection. HPV is passed from one person to another during skin-to-skin contact with an infected area of the body. HPV can be spread during sex -- including vaginal intercourse, anal intercourse, and oral sex - but sex doesn't have to occur for the infection to spread. All that is needed is for there to be skin-to-skin contact with an area of the body infected with HPV. Infection with HPV is common, and in most cases the body is able to clear the infection on its own. But in some the infection becomes chronic. Chronic infection, when it is with high-risk HPV types, can eventually cause cancer. Virtually all cervical cancers are caused by HPV infections, with just two HPV types, 16 and 18, responsible for about 70 percent of all cases. HPV also causes anal cancer, with about 85 percent of all cases caused by HPV-16. HPV types 16 and 18 have also been found to cause close to half of vaginal, vulvar, and penile cancers. HPV infections also cause cancer of the oropharynx, the soft palate, the base of the tongue, and the tonsils. In the United States, more than half of the cancers diagnosed in the oropharynx are linked to HPV-16.

75% of those who engage in sexual activity with someone who has condyloma acuminata will develop these warts within three months

Xinyi Yeow Gardasil is a quadrivalent vaccine, against HPV 16, 18 (which cause 70% of cervical cancer) and HPV 6 and 11 (which cause 90% of genital warts). That's why men should also get vaccinated with Gardasil, whereas cervarix is a bivalent vaccine, only against HPV 16 and 18 Punna Wong If two virgins marry and have a monogamous relationship are they immune to hpv / cervical cancer Chee Yong Chuan Can you get cervical cancer if you are a virgin and never had penetrative sex? It is very unlikely to get cervical cancer if you are a virgin but we do not know for sure it this protects you completely! A study done in 1952 of 13,000 nuns found no cases of cervical cancer. These studies and other more recent studies, all indicate that women are more at risk of developing cervical cancer once they have become sexually active (acquisition of oncogenic HPV strain). Can you get HPV infection being a virgin? Yes because genital contact without penetration is possible. It is possible to get HPV infections from skin contact. Human papiloma viruses are ubiquitous, can be found anywhere, even on your hands!

41. And our Journey now reaches the Lower Limbs This is a Diabetic of many years. What does he have?

Matthias Lu Zhuang Hao Necrobiosis lipoidica diabeticorum. Some of the features that points to this diagnosis are -Patient has a long history of diabetes -Usually occurs on the shins, like this patient -Irregular and indurated plaques -Red-violaceous margins -Ulceration and telengectasia is present Necrobiosis lipoidica is a necrotising skin condition that usually occurs in patients with diabetes but can also be associated with Rheumatoid Arthritis. In the former case it may be called necrobiosis lipoidica diabeticorum. NLD occurs in approximately 0.3% of the diabetic population, with the majority of sufferers being women (approximately 3:1 females to males affected). Necrobiosis lipoidica commonly occurs simultaneously with granuloma annulare and with sarcoidosis. The severity or control of diabetes in an individual does not affect who will or will not get NLD. Better maintenance of diabetes after being diagnosed with NLD will not change how quickly the NLD will resolve.

Kianseng Ng Other skin lesions in diabetics: Diabetic dermopathy Infective(Boils, candidiasis) Foot & leg ulcers Vitiligo Fat atrophy & hypertrophy Xanthomas Acanthosis nigrican Peripheral anhidrosis Chlorpropamide alcohol flush Diabetic Bullae aka Bullosis Diabeticorum Granuloma Annulare (Often mistaken for Tinea...a pitfall!) Asymptomatic ring-like (annulare) dermal papules and plaques rarely associated with Diabetes Mellitus. Histology is a granulomatous inflammation. Mostly isolated in extremities, often mistaken for Tinea infections.

Diabetic Dermopathy

Acanthosis Nigricans

42. This young patient presented with Painful lesions on her lower limbs. What is the diagnosis and what are the possible causes?

Lau Ron Hsien Erythema nodosum, a tender, shiny, erythematous nodule present most commonly over the shins. Presumed to be a hypersensitivity reaction which causes inflammation of the fat cells located under the skin or the panniculus, causing panniculitis. There are many causes to Erythema nodosum. However, for young women of child bearing age think of pregnancy. Erythema nodosum can also manifest following an infection, namely Strep throat, Mononucleosis, and Mycoplasma pneumoniae. It is also associated with TB, Hepatitis B/C, and fungal infections. Also present as a drug reaction, sulphanmides and other sulfur and hyaline based drugs are the most common culprits. Gold, Sulphonylureas, and OCPs will also cause Erythema nodosum.

Inflammatory Bowel Diseases, esp Crohn's Disease is highly associated with Erythema nodosum (Patients with Ulcerative collitis present more with Pyoderma gangrenosum). Other inflammatory disorders that can cause Erythema nodosum would be Sarcoidosis and Behcet's Disease. Malignancies can also sometimes present with Erythema nodosum, of which, the commonest would be the haematological malignancies like Hodgkin's Lymphoma and Hairy Cell Leukemia. 43. Look carefully at this patient's lower limb. What is the abnormality and what does it suggest? This picture reminds students again on the Importance of a Meticulous Physical Examination

Lau Ron Hsien The common peroneal nerve as it runs past the head of the fibula is thickened, a sign that can be seen in patients with Hansen's Disease or Leprosy. Another location to seek for nerve involvement would be the Great Auricular Nerve. Hansen's Disease, or Leprosy, is caused by Mycobacterium leprae. It is not very contagious, as it requires long term exposure to the pathogen to contract the disease. There are a few ways of classifying HD, Ridley Jopling, which has 5 classifications and the WHO, which has 2 classifications.

Ridley Jopling has classified HD as being Tuberculoid or Lepromateous, with the intermediates of Borderline Tuberculoid, Midborderline, Borderline Lepromateous. WHO classifications makes use of number of skin lesions as well as amount of bacilli found on a skin smear. They are then classified as Paucibacillary (<5 skin lesions, with no bacillli found on skin smear) and Multibacillary (>6 skin lesions, with visualisation of bacilli on smear). For the Tuberculoid types, patients usually have few skin lesions, with only a single large erthematous plaque with a well defined border and an atrophic centre. The centre of the lesion is also anaesthetic. It is also in this form of leprosy that the peripheral nerves are thickened, accompanied by some form of peripheral neuropathy. For the Lepromateous types, they usually would develop into a Leonine facies. They would start of as pale macules or nodules and infiltrate surrounding structures, often destroying cartilage and causing disfigurement. Lymphadenopathy and aseptic necrosis and osteomyelitis can also occur from bone infiltration. Patient usually present with a loss of sensation, first to temperature and then to light touch followed by the loss of pain sensation and then deep pressure. There can also be autoamputation of the fingers in patients with HD. Patients are usually treated with Dapsone, Rifampicin, and Clofazimine to prevent presence of multidrug resistant strains

44. Look at the COLOUR difference between the Hands and the Feet! And this is NOT in a cold country but Msia. What condition can explain for this?

Lau Ron Hsien Clubbing of the toes with cyanosis of the lower limbs. The upper limbs are not cyanosed; note that the Right hand does not have Clubbing while there is some Clubbing at the Left Hand! This can only point towards Differential Cyanosis and Clubbing.

This condition is unique to having Patent Ductus Arteriosus distal or near to the Left Subclavian Artery developing Eisenmenger's Syndrome as the Patent Ductus Arteriosis remained open. The mixing of deoxygenated and oxygenated blood only happens after the branches of the aorta that supplies the cephalic parts of the body, including the upper limb, has branched off. So deoxygenated blood is sent only to the lower limbs, causing the clubbing and cyanosis of the toes and the lower limbs. The proximity of the Left Subclavian artery to the PDA may account for the Clubbing seen at the Left Hand.

45. A Curtain Falling In Amaurosis fugax there is loss of vision in one eye due to a temporary lack of blood flow to the retina. It may be a sign of an impending stroke. Amaurosis fugax is a symptom of carotid artery disease. It occurs when a piece of plaque in a carotid artery breaks off and travels to the retinal artery in the eye. In people with amaurosis fugax, vision loss continues as long as the blood supply to the retinal artery is blocked. Symptoms include the sudden loss of vision in one eye. This usually lasts seconds but may last several minutes. Some patients describe the loss of vision as a 'gray or black shade coming down' over their eye. Both amaurosis fugax and transient ischemic attack (TIA) are diagnosed almost exclusively by history alone.

Amaurosis fugax in elderly patients may be the initial sign of giant cell arteritis.

Hu Mung Chee The anterior circulation of the brain supplies the retinas and the cerebral hemispheres through the internal carotid artery and its branches: the ophthalmic and the anterior and middle cerebral arteries. Temporary monocular blindness, termed amaurosis fugax is one classic manifestation caused by occlusion of the ophthalmic branch of the internal carotid artery, causing retinal ischemia. Acute painless loss of vision in one eye is most commonly due to retinal arterial or venous occlusion or giant cell arteritis. Central retinal artery or vein occlusion causes complete loss of vision in one eye, whereas branch occlusions may result in partial visual field loss. Patients should be asked about vascular risk factors and previous history of stroke or myocardial infarction. Symptom of headache in a patient aged over 50 years should raise the suspicion of giant cell arteritis. These conditions should be treated as medical emergencies for two reasons : (1) In the case of central retinal artery occlusion the prognosis for visual recovery is directly related to the promptness of treatment, and urgent referral to ophthalmology is indicated. (2) If a branch occlusion has occurred, or if the visual loss is a result of giant cell arteritis, the rest of the retina or the other eye is at risk. If the underlying conditions are not treated the patient may go on to develop complete blindness.

Kong Wah Ng The central retinal artery is an end-artery, so any occlusion of the CRA is likely to result in blindness because there is no collateral circulation 46. Barrel Chest

It is now not just the poison of cigarette smoking that causes this But also our polluted cities. A lateral film demonstrates it best. Beta Blockers are contraindicated. Can you give me the A to H list of contraindications for use of Beta Blocker? How do you differentiate between Asthma and COPD? Manissa Sufian Contraindications to using Beta Blockers A- asthma, B- heart Block, Bradycardia C- acute CCF, COPD D- diabetes, dyslipidemia E- erectile dysfunction, diseases of the extremities F- fatigue, foetus (intrauterine growth retardation) G- geriatrics H- hypotension, hypothyroidism

Kong Wah Ng Why are beta blockers contraindicated in diabetes and dyslipidaemia? Many patients with dyslipidaemia are also hypertensive Matthias Lu Zhuang Hao One of the complications of diabetes is peripheral vascular disease, leading to a decrease in blood supply to the extremities. Using beta-blockers to lower the arterial blood pressure can potentially further impair the blood supply. This might worsen the paresthesia (neuropathy), wound healing, claudication. Punna Wong In patients with poor or brittle glycemic control, the fear of hypoglycemic unawareness especially in the elderly with little neurological features, and the availability of ACEI has all made Beta Blockers a poor choice other than in patients with IHD where it is a specific indication. When there is risk of Hypo, educate recognition, and avoid Beta Blockers. Antihypertensive treatment with beta blockers has been associated with a higher incidence of diabetes, hypercholesterolemia and hypertriglyceridemia. This is worse when used in the traditional Beta Blocker-Diuretic combo. .. Features of chronic obstructive pulmonary disease (COPD) and asthma often overlap, often rendering a firm diagnosis difficult to achieve for the clinician. Asthma and chronic obstructive pulmonary disease (COPD) are characterized by a reduced rate of pulmonary airflow resulting from increased inflammation. Airway obstruction is typically fully or nearly fully reversible in patients with asthma, whereas COPD is characterized by airway obstruction that is not fully reversible. Asthma is most often associated with onset during childhood, prior atopic reactions, and a family history of atopy or asthma. In contrast, COPD is essentially unknown in children and is rare in younger adults without a history of alpha1-antitrypsin deficiency. After age 40, however, the prevalence of COPD increases substantially with aging, and the prevalence of asthma declines slightly.

In a subset of asthma patients with long-term disease, reversibility of airway obstruction is diminished (due to airway remodeling), and a disease pattern similar to COPD may ensue. When the onset of asthma occurs in the sixth or seventh decade of life, recognition is more difficult because symptoms may be similar to those of cardiac disease and COPD, and patients often accept problems as secondary to aging rather than disease. The most difficult group of patients in which to distinguish asthma and COPD are those who are current or former smokers. In smokers as young as 30 to 40 years, clinical and pathologic manifestations resembling early-stage COPD may be present. Long-term follow-up studies show that up to 23% of patients with a history of asthma develop the COPD hallmark of fixed airflow obstruction Although daily symptoms are present in only 27% of people with asthma, symptoms in COPD are more likely to be constant and progressive, reflecting the fact that airway obstruction in COPD is not due to the reversible airway but rather to structural changes and mechanical derangements with abnormal elastic recoil. Airway obstruction in asthma results from bronchial smooth muscle constriction, airway hyperreactivity to allergens and irritants, and inflammation accompanied by increased eosinophils and CD4+ lymphocytes. Airway obstruction in COPD is associated with neutrophils, CD8, cellular damage and mucus hypersecretion. Patient history should include specific queries: Age at onset of symptoms -- younger with asthma, older with COPD. Any history of atopy -- such conditions as allergy, eczema, and rhinitis are associated mainly with asthma; asthma is also suggested when symptoms are acutely triggered by exercise or exposure to cold air.

Characteristics of cough -- more likely to be dry in asthma, productive in COPD. History of smoking -- pack-years likely to be lower in asthma than in COPD. Symptom variability -- daily changes in symptom frequency or intensity are common in asthma; COPD symptoms are less variable. Note that viral upper respiratory tract infection can result in exacerbations of either condition, and this factor is therefore of little help in distinguishing asthma from COPD. However, exacerbations associated with bacterial respiratory infection are more suggestive of COPD than asthma. 47. Lobar Pneumonia

Lobar pneumonia caused by Strep Pneumonia typically present with Fever, Chills, Rigors and a terrifying cough. Blood C&S is typically positive so pls do not forget this Ix. On the CXR, consolidation is typically limited to one lobe, the Right LL being most common.

Note that the Right Heart border is intact while the R hemidiaphragm is blurred. A pleural effusion is commonly found. A lateral film is very useful to confirm the lobe. No Cavitation or Empyema (Suspect other bacteria if present). If a patient's Lobar Pneumonia Does NOT clear with treatment, what must you suspect and exclude?

Fahad Sorefan The median duration to reach clinical stability is about 3 days. Poor response to Rx beyond 3-5 days can be due to progressively worsening, non-responding or non-resolving pneumonia. Progressively worsening condition suggests: 1. Resistant pathogens, inappropriate antibiotics, nosocomial infection, (MRSA, drug resistant strep) 2. Unusual organisms (Tuberculosis, Fungal infectionscryptococcus, aspergillus, Viruses, Zoonosis) 3. Metastatic infection (parapneumonic complications, endocarditis) 4. Malignancies (bronchogenic cancer, metastasis) 5. Immunosuppressed state (comorbids - COPD, CCF, diabetes, alcoholic/hematological malignancies 6. Misdiagnosis (CCF, PE, Wegeners) 7. Complications eg abscess 8. Foreign Body Ng Hui Mien ABCDE A - Antibiotic -wrong dosage B - Bugs -medication did not cover the correct organism, Atypical Bugs C - Complications developed - Empyema or lung abscess D - Diagnosis - wrong diagnosis, could be PTB, Pulmonary Embolism, Foreign Body, Pulmonary Haemorrhage or Malignancy E - Expectation - too fast, e.g. elderly response to treatment slower

48. Red Currant Jelly A man with heavy alcohol consumption came with fever, chills and rigors. He had Red Currant Jelly sputum! What is the cause of this "Bulging Fissure" sign? An empyema followed by lung Cavitation developed after 5 days.

This is the CXR after a week

Matthias Lu Zhuang Hao -red current jelly sputum is characteristic of klebsiella lobar pneumonia, also associated with chills and rigors -the bulging fissure sign is seen with a klebsiella infection, but not limited to this. -A history of heavy alcohol consumption also puts this patient at higher risk for a klebsiella infection -Klebsiella infections tend to cause a lobar pneumonia, followed by complications such as an empyema and abscess Hu Mung Chee Classically, bulging fissure sign is associated with consolidation of the right upper lobe due to Klebsiella pneumoniae infection. Due to the tendency for Klebsiella to produce large volumes of inflammatory exudate, the involved lobe expands and exerts mass effect on the adjacent interlobar fissure. This sign has been reported in up to 30% of patients with Klebsiella pneumonia. Other less common causes of the bulging fissure sign include Hemophilus influenzae, tuberculosis, pneumococcal pneumonia, large lung abscesses, and lung neoplasms. 49. Classics

Pls give me the 5 Ts of an upper Mediastinal mass!


Reffai Syed The 5 Ts are T : thymus T : thyroid T : thoracic aorta aneurysm T : terrible lymphoma T : teratoma s : Schwanoma

This is a Classic rarely seen today!

Film Pripanapong This lateral view show radio-opacity of the ascending aorta, the sign is suggest of Luetic aortitis or extensive 'eggshell' calcification that extends from the aortic annulus through the entire thoracic aorta, which cause by syphilis.

This CXR was done on D5 after the patient had an AMI What is the Diagnosis? Wong Wai Kit a left ventricular aneurysm Kianseng Ng Ventriculography can demonstrate left ventricular aneurysm formation within 48 hours of infarction in 50% of patients who develop ventricular aneurysms. The remaining patients have evidence of aneurysm formation by 2 weeks after infarction. Over 95% of true left ventricular aneurysms reported in the English literature result from coronary artery disease and myocardial infarction. True left ventricular aneurysms also may result from trauma, Chagas' disease, or sarcoidosis. A very small number of congenital left ventricular aneurysms also have been reported and have been termed diverticula of the left ventricle.

What do you see in this amazing CXR? This chest radiograph shows air fluid level within the outline of the Pericardial sag; this is a Pneumo-haemopericardium. Other causes of an air fluid level behind the heart are hiatus hernia or achalasia cardia.

Sir William Osler

This is another Classic! We should NOT see such CXRs anymore What is your diagnosis? Barium has been swallowed to outline the Oesophagus. Pls comment on the anatomical changes seen here and its cause! Hu Mung Chee This radiograph demonstrates ski mogul heart with a prominent left atrial appendage, enlargement of right pulmonary artery and a double density over the right cardiac border. These are characteristic radiologic findings of mitral stenosis. The most common cause of mitral stenosis is rheumatic fever. Rheumatic fever is rare today. .. The Left Atrium is poorly seen from a PA view as it is at the "back" of the cardiac shadow. It may occasionally be seen as a Double shadow when enlarged or as an egg behind the heart on a penetrated view.

On a lateral view it is well seen and when enlarged the Retrocardiac space disappears! In this patient it is so big that it distorts the oesophagus that is here outlined with Barium. FROM HERE YOU CAN UNDERSTAND WHY THE MURMUR of mitral regurgitation radiates to the back and when severe can be heard at the spine or even the top of the head and neck. 50. More Classics

Describe the abnormalities here and its cause. Kianseng Ng A diagnosis of atelectasis requires the following: (I) A density, representing lung devoid of air (II) Signs indicating loss of lung volume Here there is a triangular non-homogenous opacity in the Right Upper zone. This is the lung emptied of air. Signs of Loss of Lung Volume 1.Generalized Crowding of ribs: The interspace between the ribs is narrower compared to the opposite side. 2.Movement of Hilum The right hilum is normally slightly lower than the left. Here it appears to be on the same level as the left. 3.Horizontal Fissure is displaced upwards

4.Compensatory Hyperinflation 5.Alterations in Proportion of Left and Right Lung The right lung is approximately 55% and left lung 45% in the normal, here the left appears bigger. The lower border of the opacity is probably the sharp line of the Horizontal Fissure.

RUL Collapse

LUL Collapse

RLL Collapse

LLL Collapse

Note that the R Hilum is positioned significantly Downwards compared to the Left Hilum due to loss of Lung Volume with the collapse of the Right Middle Lobe. The ribs on the Right side are also less horizontal cf the Left ribs The Right sided Intercostal spaces may also be narrowed.

RML collapse

These 2 films demonstrate the importance of a lateral film. What is the diagnosis? Film Pripanapong There is a radio-opaque consolidation on the middle lobe of the right lung suggestive of right middle lobe pneumonia. The lateral film shows that the pneumonia is in the middle lobe and not the lower lobe

Dr. Kianseng Ng, the founder of Aequanimitas

How did this happen?? What are the possible causes? Kianseng Ng Cannon Ball secondaries, from haematogenous spread. BLTKP, Bacon Lettuce & Tomato Sandwich with Ketchup and Pickles B Breast L Lung T Thyroid K kidney P Prostate Malignant Melanoma, Thyroid cancer, Breast cancer, Colorectal cancer, Renal cell cancer, Choriocarcinoma, Testicular cancer, Osteosarcoma, Ewing sarcoma, Wilms tumor, Rhabdomyosarcoma, Prostate cancer, HepatoCellular Ca. "

Punna Wong R - Renal Cell Ca I - Immunocompromised B - Breast Ca C - Colonic Ca, ChorioCa H - HepatocellularCa O - Osteosarcoma P - Prostrate Ca M - Malignant Melonoma E - Ewing's Sarcoma T - Testicular Ca

RIB CHOP MET

Today is the last day for this month of dedication to the Memory of Prof TJ Danaraj.

Medicine has given many of us a way of Living and there is NOTHING wrong with earning a decent living from an honest lifestyle "earning with the Sweat of my brow and the Strength of my Arms". However the Art of Medicine should Not become the Business of Medicine. Prof TJ Danaraj had warned us against the "Prostitution of Medicine" and he perhaps foresaw the huge conglomerates that are prevalent today and the commercialisation of this ancient Art.

It is up to the individual and his conscience as to where is this fine line to be drawn BUT as long as he remembers the Principles of Patient above Self, of Primum Non Nocere and the Hippocratic Oath, he is in safe waters. My tour of duty ends, and I thank you all for your attention.

Dr Wong Yin Onn ..

Assoc. Prof Alex Tang Thank you for your postings for this month. It has been a wonderful journey. It is appropriate that you end with the commercialisation of medical care. Medical care is now big business and whether we like it or not, it will influence the way we practice medicine in the coming years.

Assoc. Prof Keng Kiat Toh Dear Friends, I agree absolutely with Dr Wong's sentiments. We should NOT go into medicine for the money or go for the money that is in medicine. I still remember the days when my patients gave a basket of eggs and a pot of chicken curry as appreciation. In urban Singapore!!! Recently, an Indian patient embarrassed me by kneeling at my feet to kiss the ground and called me "Baba" after Sai Baba, when I told her that her CML had been cured with imatinib and the BCRABL and Ph chromosome were now cleared. Gratitude on her part and satisfaction on mine.

ADDENDUM

1. Graduation When I graduated from UM, I had high hopes that I will work as a Houseman in UH but alas I was 'banished' to a 'small' General Hospital instead. I was disappointed. I met Prof Danaraj at one of the corridors and lamented to him my 'fate'. But he had an entirely different view! 'Go and serve in the General Hospital' he said 'where you will be a Real Doctor. At UH you will just be a clerk! You will mature much faster as a doctor outside' Many years later, I was post MRCP and wanted to go back to serve in UM but the paperwork and red tape was just too slow and after a year of waiting, I gave up. An offer to work in private came and I took it. Meeting up with Prof again, I told me of my twicedisappointed hopes. Again he had a different view! He said, "The University is always there. You can always come back to Academia later on!" He was again right for 2 decades later, I am back in Academia. I had worked in Government service, Private Practise and now Academia. Hopefully this helps me to be a better teacher. If I can talk to TJD again, I will tell him,

"Sir, you are right; I did mature very fast working in a General Hospital for there Responsibility fell on me like a load of bricks. We were all at the deep end of the swimming pool after barely learning how to swim. Private practise was a completely different world with its own spectrum of diseases and patients, and Yes! The universities are still there. Thanks to Prof Khalid Kadir who invited me, I am now back in Academia. Thank you for building the Faculty of Medicine which gave me my life."
2. A Very Senior Doctor asked me to help her prepare this document.

ADVANCE HEALTH CARE DIRECTIVE


I, __________________________ IC: _______________, being of sound mind, declare that: (1) END-OF-LIFE DECISIONS:

I direct that my health care providers and others involved in my care provide, withhold, or withdraw treatment in accordance with the choice I have marked below: [___] (a) Choice NOT To Prolong Life. I do not want my life to be prolonged if (1) I have an incurable and irreversible condition that will result in my death within a relatively short time, (2) I become unconscious and, to a reasonable degree of medical certainty, I will not regain consciousness, or (3) the likely risks and burdens of treatment would outweigh the expected benefits, OR

[___] (b) Choice To Prolong Life. I want my life to be prolonged as long as possible within the limits of generally accepted health care standards. (2) RELIEF FROM PAIN: I direct that treatment for alleviation of pain or discomfort should be provided at all times even if it hastens my death: (3) PRIMARY PHYSICIAN:

I designate the following physician as my primary physician to help me carry out my wishes: _________________________________ (name of physician) OPTIONAL: If the physician I have designated above is not willing, able, or reasonably available to act as my primary physician, I designate the following physician as my alternative physician: _________________________________ (name of physician) (4) DONATION OF ORGANS AT DEATH: Upon my death:

[___] (a) I give any needed organs, tissues, or parts, OR [___] (b) I give the following organs, tissues, or parts only: ______________________________________________ [___] (c) My gift is for the following purposes:

(1) (2) (3) (4)

Transplant Therapy Research Education

In the absence of my ability to give directions regarding the use of such life-sustaining procedures, it is my intention that this declaration shall be honoured by my family and physician(s) as the final expression of my legal right to refuse medical or surgical treatment, and I accept the consequences from such refusal.

I understand the full import of this declaration and I am emotionally and mentally competent to make this declaration. I execute this declaration, as my free and voluntary act, on this _________ day of _______________, 20____, in the City of ________________, State of ________________. ______________________________________ (signature) I declare (1) that the individual who signed this advance health care directive is personally known to me, (2) that the individual signed this advance directive in my presence, (3) that the individual appears to be of sound mind and under no duress, fraud, or undue influence, (4) that I am not the individuals health care provider. I further declare that I am neither related to the patient by blood, marriage, or adoption, and, to the best of my knowledge, I am not entitled to any portion of the patient's estate upon the patient's death under a will existing when the advance directive is executed or by operation of law. Signed at __________________ on this _________ day of ______________, 20____. ________________________________________________ (name and IC of first witness) ________________________________________________ (name and IC of second witness) ..

A living will is a document that a person uses to make known his or her wishes regarding life prolonging medical treatments. It is important to have a living will as it informs your health care providers and your family about your desires for medical treatment in the event you are not able to speak for yourself. A living will describes certain life prolonging treatments. You, the declarant, indicate which treatments you do or do not want applied to you in the event you either suffer from a terminal illness or are in a permanent vegetative state. A living will does not become effective unless you are incapacitated; until then you'll be able to say what treatments you do or don't want. A living will is only used when your ultimate recovery is hopeless. This means that if you suffer a heart attack, for example, but otherwise do not have any terminal illness and are not permanently unconscious, a living will does not have any effect. You would still be resuscitated, even if you had a living will indicating that you don't want life prolonging procedures. None of these documents will do you any good if no one knows about them. Discuss with your doctor what kinds of end of life medical treatments you want. Once you've decided what it is you do or don't want, make your wishes known to your doctor and your family. This is less a Legal Instrument and more an Instrument of Communication for it communicates to your attending doctors and relatives your wishes should you be in a position where you are not able to speak for yourself. Alex Tang Advance Heath Directive also known as Advance Medical Directive (AMD) or commonly known as a living will gives instructions about what you want or do not want about medical interventions when you are no longer have the ability for decision making. I believe that this is something important to share with our patients. I also share with Christians because I believe this is part of responsible stewardship of our bodies. What is the view from other Religions?

Punna Wong The Buddhist viewpoints on the following topics are as follows: 1. Euthanasia The Rules of Discipline for Monks states "Should any monk intentionally deprive a human being of life, or look about for a knife-bringer [to help him end his life], or eulogise death, or incite [anyone] to death saying 'My good man, what need have you of this evil, difficult life? Death would be better for you than life,' or who should deliberately and purposefully in various ways eulogise death or incite [anyone] to death: he is also one who is defeated [in the religious life], he is not in communion." The scope of the precept is particularly important in the context of euthanasia, since the weight of the case for allowing euthanasia rests on the postulate that death would be better than life, especially when life seems "evil and difficult". The prohibition on taking life would therefore extend to both the assistance of suicide (including physician-assisted suicide) and euthanasia. 2. Compassion We must be aware that a commitment to the alleviation of suffering can create conflict with the principle of the inviolability of life. Compassion, for example, might lead a person to take life in order to alleviate suffering, and is one of the main grounds on which euthanasia is commonly advocated. In the Monastic Rule is a case whereby the motive for bringing about the death of the patient is stated to have been compassion for the suffering of a dying monk. Those found guilty in this situation took no direct action to terminate life, but merely suggested to a dying monk that death would be preferable to his present condition. Despite this apparently benevolent motiveie to spare a dying person unnecessary painthe judgment of the Buddha was that those involved were guilty of a breach of the precept. The essence of their wrongdoing was that they "made death their aim". It is immoral, from the Buddhist perspective, to embark on any course of action whose aim is to destroy human life, irrespective of the individual's motive.

We may therefore conclude that while compassion is always a morally good motive, it does not by itself justify whatever is done in its name. 3. Must life be preserved at all costs? Two contrasting scenarios are mentioned: "If one who is sick ceases to take food with the intention of dying when medicine and nursing care are at hand, he commits a minor offence. But in the case of a patient who has suffered a long time with a serious illness the nursing monks may become weary and turn away in despair thinking 'when will we ever cure him of this illness?' Here it is legitimate to decline food and medical care if the patient sees that the monks are worn out and his life cannot be prolonged even with intensive care." The contrast is between the person who rejects medical care with the express purpose of ending his life, and the person who resigns himself to the inevitability of death after treatment has failed and the medical resources have been exhausted. The moral distinction is that the first patient seeks death or "makes death his aim", whereas the second simply accepts the inevitability and proximity of death and rejects further treatment or nourishment as pointless. The first patient wishes to die; the second wishes to live. However, the second patient is resigned to the fact that he is beyond medical help. Buddhism does not believe there is a moral obligation to preserve life at all costs. To seek to prolong life beyond its natural span by recourse to increasingly elaborate technology when no cure or recovery is in sight is a denial of the reality of human mortality, and is seen as arising from delusion and excessive attachment.

3. What is a Professional Man? The Professional man is in essence one who provides services. But the service he provides is more than that of the labourer, even that of a skilled labourer. It is a service which wells up from the entire complex of his personality. True, some highly specialised and complex techniques might be included, but their mode of expression is given its deepest meaning by the personality of the practitioner. In a very real sense, his professional service cannot be separate from his personal being. He has no goods to sell, and no land to till; his only asset is himself. It turns out that there is no right price for his service, for what is the share of a man worth? If he does not contain the quality of integrity, then he is worthless. If he does, he is priceless. The value is either nothing or it is infinite. " So, do not set a price upon yourselves. Do not measure out your services on an apothecary's scale and say 'Only this for so much'. Do not debase yourselves by equating your souls to what they will bring in the market. Do not be a miser, hoarding your talents and abilities and knowledge, either among yourselves or in your dealings with your clients, patients or flock. Rather be reckless and spendthrift, pouring out your talent to all to whom it can be of service! Throw it away, waste it, and in the spending it can be of service. Do not keep a watchful eye lest you slip and give away a little bit of what you might have sold. Do not censor your thoughts to gain a wider audience.

Like love, talent is useful only in its expenditure, and it is never exhausted. Certain it is that man must eat, so set what price you must on your service. But never confuse the performance which is great, with the compensation be it money, power or fame, which is trivial.

This was the address by Judge E P Tuttle to the medicine graduation class at Emory University in 1957
4. Why it is so Important that Doctors MUST be well trained? Doctors are just the same as lawyers; the only difference is that lawyers merely rob you, whereas doctors rob you and kill you, too.

Anton Chekov (1860-1894) Russian writer, dramatist and physician.

5. Do you want to be a Doctor?

Dear Daryl, Your aunt has asked that I share with you my thoughts on medicine and medical education today.

Let me start by saying that I am most blessed to have trained under your Aunt, Prof Florence Wang, and I recall my time in ward 12A at University hospital under her as a most formative part of my training. Thank you Prof! It has been 3 decades since then and the world has changed. The world of Medicine and medical education has changed beyond even the most imaginative of us all had dared to dream. Some things are good and much detrimental. I will have to ask you very honestly why you wish to pursue a career in Medicine for this is very important. Most students simply have very little knowledge of the world of medicine and some very quickly become disillusioned in their clinical years. ALL my students come to me with FULL distinctions or stand at >97 percentile of their cohort; that they are brilliant is without a doubt but sadly many are NOT that motivated to serve humanity. They come into medical school because their results qualify them too, they sing the right well rehearsed song at the interview and because they think it is prestigious, they want to be a medical student! But the real world of sick humans with all the smell and excrement and drains rapidly reverse all those good feelings, and harsh reality sets in. Osler a great Physician whom your aunt and I both greatly admire said that THE VERY FIRST QUALIFICATION is a Love for Humanity. I believe that that quality is far more important than all the 'A's in your result slip. AS a GOOD doctor, you will be very important. We have plenty of doctors but not many GOOD ones. The future of many lives and families depend on what doctors do and SAY; imagine all the Good or Damage that can be done. But to be a good doctor is NOT easy. I just had a tutorial with my students and told them that the VOLUME of information that they need to read up is so MUCH that any serious student will honestly spent ALL his time doing very little but STUDY. I hope this realisation will help you understand the quality of life you will have as a medical student, or rather the lack of it.

Modern medical education has moved far beyond lecture based learning; here in the Clinical school the word 'LECTURE' is OBSCENE and students learn through SDL - Self directed learning. Its philosophy being that the advance of knowledge is so rapid that Students MUST learn to teach themselves rather than being taught formally in a Lecture Hall. This demands very matured and disciplined students for to the lazy student, SDL means Sleeping DeepLy. Are you prepared for years of being a nerd? Please remember that Doctors are first and foremost Diagnosticians those training under me MUST not only be skillful in diagnosis and management but also in social skills and leadership. When I was under your aunt, she demanded History taking so detailed and meticulous that 'War and Peace' looked like a short story in comparison. Today students speak in SMS like language, and do "Focused History Taking" instead of Detail History taking. Yes the Art of Clinical diagnosis is dying, and High Touch Medicine being replaced by High Tech Medicine. In many Once Prestigious Universities which I will not name, Final Year students now teach 3rd year students Clinical Methods!!! Your aunt will tell you that 3 decades ago, only the senior consultants in UM were allowed to teach Clinical Methods as it was considered so very2 important. Osler is certainly turning in his urn!! In your mind, do you envision yourself as a doctor spending hours talking, feeling, touching, examining the patients.... or a doctor ordering scans and more scans with nary a glance at the nameless poor sick man lying on the bed? Ask yourself that question now. Yes, the practise of Medicine is in trouble. Blatant commercialisation, rampant blood tests done without any doctors ordering or supervising, scans and probes of all kinds, are being conducted by laboratories and some doctors misguided by wants rather than needs. When your aunt was holding the fort at Ward 12A, her ward rounds will start at 8am and the whole entourage of she the Professor, Dr HC Ting the lecturer, the Med Officer, The Houseman and the medical students will follow her from bed to bed.

The Med Student or the Hseman will present the LONG DETAIL history and she will then TALK TO EVERY PATIENT WITHOUT EXCEPTION about aspects of the history and personally examine every patient to confirm or correct the juniors' findings. This of course was a long tedious process and the rounds will stretch till 1pm. But it was the Art of Clinical Diagnosis at its finest. Today, I am sad to report that rounds are spent looking at scans and reports and its a RARE occasion that the Consultant lays his hands on the sick. Sadly I am not exaggerating. I wish I were. Hope lies in every one of us doctors, present, and you the future, for the sensible management of patients; YOU remain the hope for untold numbers of patients in the future. Sadly some doctors see patients not as patients but as a disease that needs treatment that provides our source of income. This is nothing new, physicians like Osler had repeatedly cautioned against not forgetting the man behind the disease, and medicine as a calling rather than a business. The practice of medicine is an art, not a trade; a calling, not a business; a calling in which your heart will be exercised equally with your head. Often the best part of your work will have nothing to do with potions and powders, but with the exercise of an influence of the strong upon the weak, of the righteous upon the wicked, of the wise upon the foolish ~ Sir William Osler Many doctors are unhappy with their work or simply too busy to talk, console or listen. Yes thats life in the 21st century where we sms instead of talk. Many just treat the disease and completely ignore the person who has it. We hear endless tales carried by patients of doctors who grunt instead of talk, who are capable only of monosyllable conversation and who zip patients in and out of the consultation room with a speed that will make Superman jealous. I try my very best to teach my students the Art of Clinical Diagnosis but even I fear that its a losing uphill battle.

My colleagues and I here are led by Prof Khalid who is an old school clinician, and we refuse to belittle the Clinical Art despite the advancing machinery all around us. Students are amazed and impressed by all the wizardry and sometimes to my frustration fail to practise the Clinical Skills as much as I want them to. Ask yourself Daryl, what do you want? I do not want you to come to medical school and be disillusioned by what you see in the REAL MEDICAL WORLD as compared to the idealised world that we had projected. Doctors must never forget why we became doctors in the first place, and the wonder of caring for fellow humans. Some may be a bit burnt out. We had in our careers, seen and taken care of more pain and misery than most people. We saw prostitutes, drug addicts, criminals and the worst of humanity. Yes, this is another point that I must raise to you. AIDS today is the TB of your aunt's era, its everywhere! Most of my students come from very privileged families or they cannot afford the fees! Hence it is a shock to many when confronted with Vagabonds, homeless, addicts, pimps and Prostitutes. All the glamour flushes away when you realise that such folks are among the many that you will encounter daily unless you are in some Beverly Hills practise. But of course we also treated decent human beings, doting grandmothers and innocent children. No doubt, the demands on our skills and the medical-legal complexities that accompany our practise would have made even the greatest of our medical ancestors shudder. And for this your training is NOT the 5 years of medical school BUT many2 years beyond. Are you prepared for such a LONG time spend in studies and training? You had read Dr Pagal's blog and written to him, while he may be harsh and discouraging, much of what he wrote to you with regards to THE SYSTEM has some truth to it. The system is basically dictated by our political masters, a system undeniably sick, and Dr Pagal has written frankly of much of our woes that only drastic administrative tsunamis can repair.

Yes the system only makes it more painful. Long waits for postgraduate training positions is the norm now. Regulations as thick as a medical text. Pharmacies selling controlled medicines like sweets. To change that will require one of us to be the next Prof Virchow, plunging head on into politics to make a difference. In school we rose above the hoard, we were the cream de la cream, and yes I can testify to that in my students. Some are so brilliant they frighten me. We obtained results the envy of most and we strove with pride to enter medical school. Could the same students have done well in other fields? Of course they will shine too!! In medical school you will work like ants on a long march. You will stare at slides till you see mitochondria in your dreams and memorised volumes of facts and figures. Some of my students are NOT prepared for this; they thought that it will be a smooth passage and when they hear this old Professor here telling them to read XYZ, they simply turn off and zone away. Are you ready for a student life that has NO LIFE?? I take them for tough postings in Internal Medicine, I am among the strictest examiners, I expect the students to flourish in the drudgery of crowded wards, to work till hypoglycaemic on medicine rounds, to perform every procedure required in the book, and of course to pass their exams reasonably well. I expect them to look at X-rays until their eyes turned red. And do I have such students? Yes, I am blessed that many such students have walked the wards with me. And I am proud of them. And I know they will overcome every obstacle invented by our political masters to excel in their fields because they have the fundamental quality to be a good doctor; their love of Humanity and of the Art of Medicine. You will finally graduate and become a houseofficer. Yes the field is as packed as sardines from Msia to UK to Australia, Dr Pagal is NOT exaggerating when he says that jobs will be difficult to come by soon. But if you are GOOD, you will be wanted! Its TOUGH I do not deny and TOUGHER as the years pass by, BUT again I emphasize, if you are good, you will be wanted for the good doctor is a rare specimen.

A few years later, you will pick a specialty, from paediatrics to surgery, family medicine to cardiology, internal medicine to radiology, and shuffled off to more clinics, rounds grand or otherwise, work and studies. And MORE EXAMs. Girlfriend? Oh what Girlfriend!!! In the 80s, new diseases appeared. I still recall with trepidation managing the very first patient admitted to our hospital with AIDS. We knew very little but feared a lot. As registrar, it was my duty to examine him. But again we survived. And learned. Now at almost every bedside teaching, I see patients with HIV. In your generation, you will see many more new ones, from Avian flu number-something2 to any new thing that crosses species as humanity ravish the environment. Ready for that? Your youthful enthusiasm and dedication will push you through the initial years. As young doctors, werent we incredible then, if only because we came back to the wards night after night, day after day for emergencies, calls, rounds or simply a tissued drip. Bengkak the nurse will phone and we leave our dinner to struggle with chemo-wrecked veins for IV access. Did you know that your aunt will reprimand the ladies for coming to the wards in pants? It is NOT acceptable then but now almost all my female students come in pants! OOPs sorry Prof for leaking such old tales! We as doctors try to hold onto our commitment to heal, that wonderful calling. Then we see another world the realm of the business of medicine, where every disease is a case to be investigated. A very senior Professor tells me she had seen doctors ordering investigations before even taking a history! Here is also the world of the grunting and monosyllable doctor, the superman of 2 minute consultations and management. True, the superb rare genius of a diagnostician may well have obtained all the data that he needed sub- 2 minutes, but the poor human called the patient needed at least 6 minutes of compassionate conversation.

(By the way, that is why the OSCE exam is 6 minutes long, or SHORT from your examinee viewpoint.. now you know how the patient will feel when the consultation is even shorter than this!). Recall that the only reason the woman in labour remembered the attending medical student is because they held her hand while she screamed. We are the descendants of Aescalapius, the inheritors of all that is noble in the Hippocratic oath. We may fail to change the ideas of many doctors however we may preach from some illusive high moral ground. But as individuals can YOU NOT lose that ideal? If you can, welcome to the world of medicine and modern medical education. You will thrive no matter what! If not, seriously reconsider another vocation. If we tell our students that the learning of medicine is through their apprenticeship to us their seniors, then we better be sure that we are good role models. Your aunt was a superb clinician and diagnostician and had inspired many of us. As a future doctor, you can be the hope of the febrile, the breathless and the pregnant. Do not let what you see in the misadventures of some doctors discourage you. Instead let them be teachers to you for you now know what you do NOT want to be like. When a doctor have taken medicine to be a business or trade, he will ask what are his achievements material success, cars, wealth, etc.. When a doctor has taken medicine to be a calling, he will ask what has he become his character. I hope we doctors can discern and reflect on what we have become in the practice of medicine, and teach all our young charges, delivering them safely through the long 5 years of protracted labour into a reasonably sane medical world. Daryl, I hope I have helped you somehow. Your aunt, Prof Florence has taught me much, this is my little tribute in return, my Bunga Emas to her. Thank you Associate Professor Wong YO.

No operation should be carried out unless absolutely necessary . . . nor should a surgeon operate unless he would undergo the same operation himself in similar circumstances" John Hunter, Father of Modern Surgery.

John Hunter is considered one of the greatest anatomists of all time and the founder of experimental pathology in England. Hunter put the practice of surgery on a scientific foundation and laid the framework for the twentieth century developments. His saying "Don't think, try the experiment" has inspired generations of doctors. Tan Chow Wei No operation should be carried out unless absolutely necessary" How true! Aren't we seeing fall too many unnecessary LSCS, angios, laparoscopies, etc?

Tuesday May 1, 2012

Testament to TJ Danaraj
By ROUWEN LIN star2@thetar.com.my

A new book celebrates the man who founded the countrys first medical faculty. HES just around the corner, make yourself scarce! The foundation dean of Universiti Malayas medical faculty had a reputation that so preceded him that students would promptly scurry off in the opposite direction from where he was coming!

The late Tan Sri Prof Dr Thamboo John Danaraj (1914-1996) known fondly as Danny Boy or TJ was renowned for being a formidable figure; a strict disciplinarian who demanded nothing but the best from his students and staff. A great believer in the value of bedside study, TJ often made his way into hospital wards to provide hands-on teaching moments. Former health ministry director-general Tan Sri Dr Ismail Merican recalls in a new book that one of his classmates was so intimidated by TJ that he went to great lengths to get out of these relentless grilling sessions: The moment TJ walked in, SMK would walk out with measured steps, looking resolute and determined to unravel the mystery of the urine specimen clutched intimately in his hand.

That way, he could escape the drilling and grilling, and at the same time earn TJs admiration for doing ward tests in the side laboratory.... The sheer speed of obtaining a urine specimen each time TJ walked in made most of us wonder whether the specimen came from his very own bladder! Dr Ismail writes in TJ Danaraj: Doctor And Teacher Extraordinaire, a book compiled and written by Puan Sri Dr Wong Hee Ong. TJs perseverance in his quest for excellence in medical education certainly knew no bounds: Here was a man who, in response to a dire shortage of doctors in Malaya, established the medical faculty and hospital of Universiti Malaya in Kuala Lumpur from scratch. (Up to the 1950s, the only medical school in Malaya and Singapore was the King Edward VII College of Medicine, which later became the Faculty of Medicine in Universiti Malaya, Singapore.) TJ said he couldnt wait, he was really in a hurry. He said we needed medical schools, that we didnt have enough doctors in the country, says Dr Wong during a chat in Kuala Lumpur recently about the book, which will be launched on Saturday. Anxious to produce competent doctors as quickly as possible, TJ decided that the first intake of students (in 1963) would start their premedical year at the universitys science faculty even as the medical centre was being built. The faculty was officially opened in 1965.

Explaining why she decided to publish this book 16 years after TJs death, Puan Sri Dr Wong Hee Ong says, At each reunion, students would write comments in a book TJ brought along with him. This, and letters written to him that he kept, gave me the idea of putting in writing how people saw him. Moreover, many students are now in high positions practising and teaching medical students and it is good after so many years to hear their views of how they were taught by TJ as students. Even before the first batch of students graduated, the faculty commanded admiration when the General Medical Council in Britain, following an inspection, decided that the schools medical degree would be recognised and graduates could practice in Britain and all Commonwealth countries. (This recognition was withdrawn by the council in 1990.) That first batch of graduates in 1969 and subsequent batches achieved 100% passes in the Educational Commission for Foreign Medical Graduates (ECFMG) examinations worldwide. (The ECFMG assesses the readiness of international medical graduates to enter residency or fellowship programmes in the United States that are accredited by the Accreditation Council for Graduate Medical Education.) The director from the examination centre wrote and asked (TJ) what the faculty was feeding the students! It was very impressive, especially for a new school, says Dr Wong, who is currently councillor and chief surveyor of the Malaysian Society of Quality in Health and is involved with hospital accreditation surveys. She was one of TJs medical students, and in later years, his colleague and spouse. TJs first marriage was to Prof Dr Winifred Lewis, who died in 1977. Awe-inspiring figure TJ was known to be tireless, strict and demanding. A dedicated teacher, he had no qualms about ticking students off as he saw fit and pushing them to deliver their best. His bark was worse than his bite, says Dr Wong, in reference to his reputation. Invariably the initial fear on first encounter turned into expressions of respect, affection and friendship, she adds. It seems that his students agree.

Former prime minister Tun Dr Mahathir Mohamad, who joined the King Edward VII College of Medicine in Singapore as a student in 1947, the same year as TJ joined the institution as a tutor, says in the books foreword that long before he was in his clinical years, TJs name was already well-known throughout the college. He was known for his strictness and dedication to teaching. He was tireless. It was with some trepidation that I attended his lectures and clinical presentations in my fourth year. I wasnt the best of students and I did not expect to follow the presentations as well as the others. To my surprise, I found his talks and presentations very lucid and easy no only to understand but to remember....

TJ and his favourite dog, Hagen. At TJs first posting, he was the districts only doctor, so he wore many hats including that of veterinarian! Personally I owe much to his teaching methods for I found them useful for solving (diagnosing) many political, economic and social problems throughout my career as prime minister of Malaysia. His passing away was a great loss to the country, writes Dr Mahathir. Monash University Malaysias school of medicine and health sciences head Prof Datuk Dr Anuar Zaini, a former student of TJs, says in an email interview that when he entered medical school, the senior students gleefully regaled him with horror stories of TJ being a stern, no-nonsense sort of guy who could swallow you up easily.

That gave me the first impression that life in medical school was going to be tough and frightening. I first met TJ when he addressed the class to lecture us on what it takes to be a medical student and then a doctor, if we survived the course. He was awesome, distinguished, knowledgeable, experienced and confident. He was assertive and terribly engaging. His diminutive stature was insignificant everything about him was big and powerful! However, he had a characteristic smile that is difficult to describe but that reminded me of my dad and gave me the impression that within him, he had a good heart. That impression has never left me till this day, Prof Anuar says. TJ made a similar impression on Sime Darby Medical Centre cardiologist Dr Kannan when he was a student. He was a strict disciplinarian, he was intimidating, yes. But my impression of him changed years later when he helped me prepare for postgraduate exams, and especially after I started working as a lecturer in medicine. I have been blessed to have had, and still have, many good teachers. TJ was one of the best. His teaching skills were second to none and I still remember his clear, concise delivery of salient points. I was just an average student but he could clarify the most difficult problems in just a few important sentences. He was a caring man, who wanted all his students to achieve their full potential and contribute to society in general, and medical education in particular, he says. Dr Wong explains that TJs straightforward nature resulted in him stepping on many toes along the way of ensuring his faculty produced the best possible doctors. Hes quite fiery and appears very stern to most people who first meet him, but the students really were his life. Without them, he would have been completely lost. And when you got to know him, you would see that he only had their best interest at heart. Deep down, he had a really soft heart, says Dr Wong, adding that TJ had a huge collection of poetry books and he used to tramp around Frasers Hill in Pahang with a pair of binoculars in hand to bird-watch. Linda Yip recollects the first time she met TJ in his house: I was in Form Six and studying in the same school as TJs daughter, Shanta. I remember being scared when I went over to their house for the first time. Id heard tales about how all the students were scared of him, how strict he was.

But when I was there, I realised that there was really no need to be frightened of him. His son was studying abroad, and in the all-female household, TJ really wasnt the most formidable character there! Over time, Yip says she became like a surrogate daughter, frequently visiting Shanta at the house and going on holidays with the family. The human touch Despite his commanding presence among his students, TJ was known to be particularly sympathetic to the needs of patients; he preached the importance of practising humanistic medicine treating the patient not as an isolated case or a set of symptoms but as a fellow human being. Not surprisingly, the medical school curriculum he planned was based on the philosophy that medicine is not only the art of healing but also the art of prevention and rehabilitation. Prof Anuar comments, He was a man who believed and practised the gentle art of healing. His message to us was always to understand in detail the problems presented by patients and not hastily embark into fanciful investigations and interventions. Dr Wong says that TJ had a wealth of clinical experience that he was keen to share. He always emphasised the importance of listening to the patient, because the patient is telling you a story. Diagnosing a patient is like a detective story you put your clues together, come up with possible ideas about what the patient is suffering from and then decide what to do to confirm this. TJ was very organised, he kept very meticulous notes. Following these logical steps to arrive at a diagnosis was crucial in an era without CT (computed tomography) scans and MRI (magnetic resonance imaging) machines. Medical training in those times placed great emphasis on using the five senses to make a diagnosis. This is equally if not more important these days with advancements in technology, which should be used to assist and confirm the clinical diagnosis (not make the diagnosis). Sadly, this is now a dying art, Dr Wong says. Prof Anuar recollects an incident during his student days when he was the houseman on-call and had to stay up all night to prepare for a morning briefing with TJ.

In the morning, he flipped through my notes and saw that I had spelled vomiting with double t. I must have inadvertently written it while trying to fight my droopy eyes! Laddie, what new pathology have you noted down here? he asked me. However, the stress and embarrassment was short lived when he openly complimented me on the good diagrams and detailed description that I had done and instructed the rest of the housemen to follow. Naturally, I was hounded by my colleagues for allegedly trying to raise standards! It became a norm for every one of us to make sure we were prompt and proper every time he was scheduled to take the morning reports, Prof Anuar relates. Until today, Prof Anuar says, he appreciates the reasons why TJ placed so much importance on accurate and systematic clinical documentation. We need to listen properly and be meticulous in taking the patients history to formulate a rational clinical approach before examining them. Nowadays, many of our younger colleagues are either not exposed to such finer arts of medical knowledge or just cannot be bothered. It is sad indeed to see such a trend, particularly so when we know this is can be partly attributed to the myriad of medical programmes sprouting out like mushrooms all over the country, he says. TJ was apparently not keen on the idea of private practice. He foresaw and warned about transposing local practice into big business, where the driving force will inevitably shift from providing a service to the people to being a money making venture. He believed that medicine should not discriminate on account of race, religion, wealth, status or any criterion between one patient and another, between one human being and another. Sime Darbys Dr Kannan remembers that among the many things he learned from TJ, one of the most important is the honest practice of medicine. (This) is all the more relevant in this day and age of commercialisation. He once told me that private practice is the prostitution of medicine! He forms an important part of my conscience as I go about caring for my patients needs, and I have tried to emulate his teaching style in my work, says Dr Kannan. Humble beginnings TJs contribution to medical education in the country seems especially formidable considering his humble beginnings.

Born the second son of a civil servant in Ipoh in 1914, TJ attended the Anglo Chinese Boys School and finished school when he was 15 years old. Two years too young for admission to medical school, he taught at the Teluk Anson Methodist Boys School in Perak before starting at the King Edward VII College of Medicine in Singapore in 1932. His scholarship money of $20 (Straits dollars) per month from the state was insufficient to cover his tuition fees and books, so he took to donating blood, for which he was paid. Still, there came a time where he was suspended for a term during his fourth year in medical school for not being able to pay the fees. But he let nothing faze him. Out of the 20 students in the same batch as TJ, only two graduated within the minimum period of six years. TJ was one of them. Graduating during the economic slump in 1938 (the world was still in the throes of the Great Depression that had started when American banks failed), he was happy to be drawing a monthly salary of $75 for working at the pathology department of the General Hospital in Singapore; many others worked for free just to gain experience. The following year, TJ was posted to Dungun, Terengganu. Being the only doctor in the district, he became a man of all trades, functioning as a health officer, obstetrician, physician, surgeon and even veterinarian! After a series of postings all over the country, he eventually embarked on his postgraduate studies in Britain and the United States. Conferred the title of emeritus professor upon his retirement as dean of the medical faculty of the University of Malaya in Kuala Lumpur in 1975, TJ continued to teach and serve at the university hospital as an honorary consultant. He also accepted an invitation from the King Abdulaziz University in Jeddah, Saudi Arabia, to advise on and assist in the setting up of its new medical school. In the three years he spent there, he helped formulate a curriculum for the medical centre, taught and trained students in their clinical years, and was involved in planning for the universitys new health science centre. TJs career in medicine spanned over 50 years of his life, during which he touched the lives of thousands of medical students in the country as well as abroad.

Saying goodbye Diagnosed with Parkinsons in 1992, TJ progressed from having trouble walking to using a cane, then a walker, and finally, a wheelchair. He stopped teaching in 1993. How did he deal with it? He lived with it, he carried on as best as he could, says Dr Wong, adding, He even drove his little yellow car to teach until such a time when he couldnt do it any longer. He continued reading a lot Robert Browning was his favourite poet. It was very frustrating for him because even though he was mentally alert, his speech was affected and people sometimes couldnt understand what he was trying to say. In a speech delivered at a farewell dinner given by the Medical Alumni Association of the university in 1977, TJ said, I ask for no honour. To see my students achieve professional and academic distinction is honour enough.... I have had a deep desire to help, to do something, to build. If I have succeeded a little, it is through good fortune and the cooperation of many. On my tombstone you can say, He tried. And if you are generous, you might add the words, very hard. TJ died in 1996, at age 82. He certainly tried, very hard. And he made his mark. .

Prof Wong Hee Ong

1st May 2012

Thank you also for devoting the month of April to honouring TJ. He would have been proud of you. H O Wong

Dr Mario Yoong: I have only joined Aequanimitas for 1 month and in that short month, I had the most exciting and challenging time which I had not experienced for a long, long time (decades). Having served as a houseman in 1965 (some 47 years ago) and had been in private medical practice for more than 4 decades had placed me in a terrible disadvantageous position. For many years I had been exposed to the usual day to day mundane routine for seeing and managing coughs and colds, URTI, hypertension, diabetes, doing simple dressings, toilet and suturing of simple wounds, seeing patients with depression, neurosis and anxiety etc. Yes medicine had been changing and developing at such a rapid pace with so many new technological advances such ultrasounds, CT Scans, MRI, laparoscopy, interventional cardiology etc which most of the GPs do not have the opportunity to learn about and be exposed to as we go about our daily routine. Yes, we had been disadvantaged and the degree of which become more obvious when we are finally placed in this type of situation. Indeed, joining Aequanimitas Medical Group had placed me in that type of situation and suddenly I realized how backward, outdated and inadequate I am. What we had learnt as a medical student, especially, the basics such taking a good history, using standard clinical methods in our examination, communicating with our patient and their family, using a lot of common sense are still the bread and butter of medical practice, especially a GP practice. A strong foundation of the basics will still hold us in good stead in general practice. That however is just not good enough anymore. We must constantly be aware of the major changes in medicine and must keep ourselves up to date with such changes. We may not be required to know the intricacies of such changes but we must at least know that these options are available so as to better advise our patients whenever the need arises.

We must always be mindful that we are the physicians of first contact and our patients usually depend on us for guidance and advice. We will be doing a great disservice to them if we are not able to provide the proper and adequate guidance and advice. In this short span of 1 month, I had learnt so much and had the opportunity to revise much of my medical knowledge, some of which had been almost forgotten, if not for the golden opportunity provided here. Prof Wong Yin Onn had done a tremendous job of teaching and most of my learning came from his input. But along the way I also learnt so much more from the other participants and contributors such Drs Patrick Cheah, Hu Mung Chee, Ho Loon Shin, Prof Ng Kong Wah, Ng Kian Seng and even the many good and active students, especially with their many Mnemonics. I have learnt about Differential Clubbing which I had never seen before in my long medical career. I revised on Von Recklinghausen's Disease (neurofibromatosis), which we still see even as a GP. I revised on STD, learnt about Gowers sign, seen some exotic huge scrotum elephantiasis, skin signs such splinter hemorrhages, Osler nodes and Janeway lesions, and so much more. GPs are usually disadvantaged with reading and interpretation of ECGs and Xrays (including CT Scans and IMR). We are not exposed enough and this where we hope to pick up a little from the teachers and the students. Yes some of the students are really good and we do learn from them. Along the way, I had the opportunity to look into and revise my knowledge of medical history, revise on the Hippocratic Oath, the Caput Medusa Sign, Primum Non Nocere etc. I also tried to be involved and participate in the discussion. In a small way, I also tried to contribute a little towards the website design by adding new graphics and help to design our new AeQ Logo something I am pretty good at. Yes it had been an exciting and challenging one-month, a little stressful at times and even loss some sleep during the preparation of designs etc. The most important thing is that we learn and we must enjoy learning. Continuing medical education is a life-long process and we must never stop.

The Aequanimitas medical Group is just the place to be in to enjoy the opportunity of learning medicine and still have fun. Besides that it is a fantastic forum to get to know some great doctors, teachers and students. We socialize, make friends, we learn and we share that is Aequanimitas. The Aequanimitas formula must be the right formula how else can we have over 1,100 members in just 2 months. I am so glad to have joined this group and have become a small part of it. Thank you." Warmest regards,

Mario Yoong

Kianseng Ng to all at Aequanimitas I am, on 30 April 2012, at 5 am, writing this tailpiece to close the Emeritus Professor TJ Danaraj Month on Aequanimitas. April has been a whirlwind. And more.

Medical blogs like Aequanimitas are nothing new; the net has shoals of medical e-sites. But designating a month to honor a giant of a man is probably something novel. And getting his protg, Assoc. Professor Wong Yin Onn of Monash to teach interactively on Aequanimitas is perhaps the honey in the rock. A multitude waited to sit at his feet, to catch the pearls that fall. And so April has been a frenzied month of teaching (for Prof Wong), learning (for people like myself) and sharing (for so many others) within an e-family which now numbers more than a thousand. April has been a mountain experience for me but the summit encounter was not in April! (Prof Wong had started posting as early as March.) The moment of transfiguration was reading, in March, A Letter to Daryl, a letter written by Assoc. Professor Wong Yin Onn to the nephew of Professor Florence Wang. This letter is compulsory reading for all doctors-wannabe and doctors-already-are who dream of their off-springs taking after them, not taking roads less travelled. This letter is now promoted (Facebook language) from Post to Editorial. And because a great number of members joined after it was posted, it will be re-featured in Aequanimitas in May as our First Editorial. Aequanimitas will now not only feature structured teaching but also Editorials, personal thoughts that may well become compass points for many. What have we been promoting and teaching in this month of April? You have to read Assoc. Professor Wong for the answer; you have to read beyond the lines, you have to read his heart beats. I believe he preaches only one thing, that doctors hold high the staff and serpent of Aesculapius so that all who lift up their eyes to gaze upon these emblems will be healed and purged of the poison in their bodies. I could wax lyrical about what Prof Wong is teaching but words will not suffice. When words or prose fails, I turn to poetry. So here are three poems, imagine three persons (a medical student, a doctor and a patient) speaking to you of those gentle influences (not "greater knowledge") that characterize Assoc. Professor Wong Yin Onn and his teachings.

And then, for those who knew Emeritus Professor T J Danaraj, ruminate on how Prof Wong reflects so faithfully the Teacher Extraordinare who many years ago took a young Yin Onn under his wings. In April 2012, Prof TJ Danaraj is till teaching... The Student Speaks... Neuroanatomy Summer The even flow of neuronal pattern was visible on the green screen, as tiny electrodes were placed directly into portions of the cats brain. I was given the job of sectioning incremental portions of the brain and measuring the cats response, reflexes, gait, etc., until it became decerebrate, with no higher brain function at all. Then the cat was rigid, fully stretched out and unresponsive. What was remarkable the professor said, was that I was able to take this cat through twelve step-wise operation (surely the basis of a paper) and keep the cat intact. Later on I saw a man stretched out like that and rigid with tubes in his nose and penis. Someone more important than me was writing a paper, I thought. Marc J. Straus The Doctor Speaks Line Drive I had come from a dying mans room. What I said, addressed the cancer eroding his brain, how his right side would become limp, he would slowly slip into a coma. And, I pointed out the silver shadow on black film from the foot of his bed, staying at beds length from him and his widow-to-be.

And, I said too little, too quickly for the quivering ears and lips where death was digging in, and, I forgot to touch or be touched. On the way home, I pass a schoolyard. It was a fine day, the light scattered in shafts on the dark brown earth of the baseball diamond. Clover grew in clumps by the side of the field, white tendrils pulling in the bees. I stood outside the chain link fence watching the game in progress. Everything was shouting, Hallelujah, alive, alive! A mother cat cuffed her kittens into line, taught them to balance on the schoolyard big-toy and old rethreads hooked one to another, swaying gently in the breeze. I jumped the fence to get closer to the game when the batter hit a line drive into the pitchers stomach. He dropped like a stone but was, in moments, surrounded by Billy and Peter and Rob and the first baseman holding his head, his sobbing, coaxing him to breathe, youll live, it will be o.k. I knew I must go back to the man dying in white linen and say nothing more than the warmth of my hand in his. Arthur Ginsberg The Patient Speaks Therapy You attribute my recovery To nor trip tylineIts effect on neurotransmitters, on the a myg dala. You barely nod towards your worthInsisting on blood levels, On a therapeutic dose.

While I credit half our success To the pear tree blossoming white Beyond your left shoulder, To the wisteriaIts pink flowers hanging Lush and fragrant Over the portico, To the warmth of your hand John Wright All three poems were taken from Blood & Bone, Poems By Physicians, 1998. With Warm Regards to all AeQuirians Kianseng

Epilogue

Now this is not the end of our studies in Medicine. It is not even the beginning of the end but it is, perhaps, merely the end of the beginning. Our studies in Medicine is but a lifelong quest for Perfection in the Art; this is why we call our work the Practise of Medicine for we are all students, we are all practicing to be better no matter how junior or senior we are in years.

Das könnte Ihnen auch gefallen